Вы находитесь на странице: 1из 156

Published by

AMT PUBLISHiNG

Mathernafics Trust
University of Canberra ACT 2601
AUSTRALIA

Copyright C2001 AMT Publishing

Telephone: +61 2 6201 5137


AMTT Limited ACN 083 950 341
National Library of Australia Card Number and ISSN
Australian Mathematics Trust Enrichment Series 1SSN 1 326-0170
101 Problems in Algebra iSBN 1 876420 12 X
THE AUSTRALIAN MATHEMATICS TRUST

ENRiCHMENT SERIES

MiTT E EJ

Chairman GRAHAM H POLLARD, Canberra AUSI'RALiA


Editor PETER J TAYLOR, Canberra AUsTRALiA

WARREN J ATKiNs, Canberra AUSTRALIA


Eo J BARBEAU, Toronto CANADA
GEORGE BERZSENYI, Terra Haute USA
RON DUNKLEY, Waterloo CANADA
WALTER E MIENTKA, lincoln USA
N1KOLAY K0NsrANT1N0v, Moscow RussiA
ANDY Liii, Edmonton CANADA

JORDAN B TABOV, Sofia BULGARiA


JOHN WEBB, Cape Town SouFH AFRiCA

The books in this series are selected for their motivating, interesting
and stimulating sets of quality problems, with a lucid expository style
in their solutions. Typically, the problems have occurred in either
national or international contests at the secondary school level.

They are intended to be sufficiently detailed at an elementary level


for the mathematically inclined or interested to understand but, at
the same time, be interesting and sometimes challenging to the
undergraduate and the more advanced mathematician. lt is believed
that these mathematics competition problems are a positive
influence on the learning and enrichment of mathematics.
PREFACE

This book contains one hundred highly rated problems used in the train-
ing and testing of the USA international Mathematical Olympiad (IMO)
team. It is not a collection of one hundred very difficult, impenetrable
questions. Instead, the book gradually builds students' algebraic skills
and techniques. This work aims to broaden students' view of mathemat-
ics and better prepare them for possible participation in various mathe-
matical competitions. It provides in-depth enrichment in important areas
of algebra by reorganizing and enhancing students' problem-solving tac-
tics and strategies. The book further stimulates students' interest for
future study of mathematics.
INTRODUCTION

In the United States of America, the selection process Leading to par-


ticipation in the International Mathematical Olympiad (IMO) consists
of a series of national contests called the American Mathematics Con-
test 10 (AMC 10), the American Mathematics Contest 12 (AMC 12),
the American Invitational Mathematics Examination(AIME), and the
United States of America Mathematical Olympiad (USAMO). Partici-
pation in the AIME and the USAMO is by invitation only, based on
performance in the preceding exams of the sequence. The Mathemati-
cal Olympiad Summer Program (MOSP) is a four-week. intense train-
ing of 24-30 very promising students who have risen to the top of the
American Mathematics Competitions. The six students representing the
United States of America in the IMO are selected on the basis of their
USAMO scores and further IMO-type testing that takes place during
MOSP. Throughout MOSP, full days of classes and extensive problem
sets give students thorough preparation in several important areas of
mathematics. These topics include combinatorial arguments and identi-
ties, generating functions, graph theory. recursive relations, telescoping
sums and products, probability, number theory, polynomials, theory of
equations. complex numbers in geometry, algorithmic proofs, combinato-
rial and advanced geometry, functional equations and classical inequali-
ties.
Olympiad-style exams consist of several challenging essay problems. Cor-
rect solutions often require deep analysis and careful argument. Olym-
piad questions can seem impenetrable to the novice, yet most can he
solved with elementary high school mathematics techniques, cleverly ap-
plied.
Here is some advice for students who attempt the problems that follow.
Take your time! Very few contestants can solve all the given prob-
lems.

Try to make connections between problems. A very important


theme of this work is; all important techniques and ideas featured
in the book appear more than once!
Olympiad problems don't "crack" immediately. Be patient. Try
different approaches. Experiment with simple cases. In some cases,
working backward from the desired result is helpful.
Even if you can solve a problem, do read the solutions. They may
contain some ideas that did not occur in your solutions, and they
VIII

may discuss strategic and tactical approaches that can be used else-
where. The formal solutions are also models of elegant presenta-
tion that you should emulate, but they often obscure the torturous
process of investigation, false starts, inspiration and attention to
detail that led to them. When you read the solutions, try to re-
construct the thinking that went into them. Ask yourself. "What
were the key ideas?" 4How can I apply these ideas further?"
Go back to the original problem later, and see if you can solve it
in a different way. Many of the problems have multiple solutions,
but not all are outlined here.
All terms in boldface are defined in the Glossary. Use the glossary
and the reading list to further your mathematical education.
Meaningful problem solving takes practice. Don't get discouraged
if you have trouble at first. For additional practice, use the books
on the reading list.
ACKNOWLEDGEM ENTS

Thanks to Tiankai Liu who helped in proof reading and preparing solu-

Many problems are either inspired by or fixed from mathematical contests


in different countries and from the following journals:
High-School Mathematics, China
Revista Maternatic Romania
Kvant, Russia

We did our best cite all the original sources of the problems in the solu-
tion part. We express our deepest appreciation to the original proposers
of the problems.
ABBREVIATIONS AND NOTATIONS

Abbreviations
AHSME American High School Mathematics
Examination
AIME American Invitational Mathematics
Examination
AMC1O American Mathematics Contest 10
AMC12 American Mathematics Contest 12,
which replaces AIISME
ARML American Regional Mathematics League
IMO International J\Iathematical Olympiad
USAMO United States of America Mathematical Olympiad
MOSP Mathematical Olympiad Summer Program
Putnam The William Lowell Putnam Mathematical
Competition
St. Petersburg St. Petersburg (Leningrad) Mathematical
Olympiad

Notations for Numerical Sets and Fields


Z the set of integers
the set of integers modulo n
N the set of positive integers
N0 the set of nonnegative integers
Q the set of rational numbers
Q+ the set of positive rational numbers
Q the set of nonnegative rational numbers
the set of n-tuples of rational numbers
R the set of real numbers
IR+ the set of positive real numbers
R the set of nonnegative real numbers
R'2 the set of n-tuples of real numbers
C the set of complex numbers
C 1

PREFACE vii
INTRODUCTION ix

ACKNOWLEDGEMENTS xi

ABBREViATiONS AND NOTATiONS xiii

1. INTRODUCTORY PROBLEMS 1

2. ADVANCED PROBLEMS 13

3. SOLUTIONS TO INTRODUCTORY PROBLEMS 27

4. SOLUTIONS TO ADVANCED PROBLEMS 65

GLOSSARY 131

FURTHER READING 137


iNTRODUCTORY PROBLEMS
1 INTRODUCTORY PROBLEMS

problem 1

Let a. b. and c be real and positive parameters. Solve the equation


CX-r ax = ax + bi + - cx.

Problem 2
Find the genera! terni of the sequence by = 3. x1 = and

for alln E N.

Problem 3
Let x1. x2 x,, be a sequence of integers such that
(i) n:

(ii) x1 + x2 + + = 19:

(iii) =99.
Determine the minimum aiid niaxiniuni possible values of

Problem 4
The function f. defined by
ax + h
1(x)
= c.r+ d
where a. b. c. and d are nonzero real nunibers. has the j)rOperties

f(19) = 19. f(97) = 97. and f(f(x)) =


forall values of x. eXCCl)t --

Find range of f,
2 1. Introductory Problems

Problem 5
Prove that
(ab)2 < a+b (ab)2
8a 2 8b
for all a b> 0.

Problem 6
Several (at least two) nonzero numbers are written on a board. One may
erase any two numbers. say a and b, and then write the numbers a +
and b instead.
Prove that the set of numbers on the board, after any number of the
preceding operations, cannot coincide with the initial set.

Problem 7
The polynomial

may be written in the form

ao + a1y + a2y2 + a15y16 + a17y17,

where y = x + 1 and are constants.


Find a2.

Problem 8
Let a, b, and c be distinct nonzero real numbers such that

a+-=b+-c
1

b
1 1

a
Prove that = 1.

Problem 9
Find polynomials 1(x), g(x), and h(x), if they exist, such that for all x,

( 1 ifx<1
3x+2 ifi
2x+2 ifx>0.
i. Introductory Problems

Problem 10
Find all real numbers x for which
8x+27x 7
12x + 18x

Problem 11
Find the least positive integer m such that

(2n'\
I I

for all positive integers n.

Problem 12
Let a, b, c, d, and e be positive integers such that

Find the maximum possible value of max{a, b, c, d, e}.

Problem 13
Evaluate
3 4 2001
1!+2!+3! 1999!+2000!+2001!

Problem 14
Letx= 1, a ER.
Find all possible values of x.

Problem 15
Find all real numbers x for which

+ lix + 12T = 13X + 14x.


4 1. Introductory Problems

Problem 16
Let f N x N N be a function such that f(1, 1) = 2,

f(rn + 1. n) = f(rn. n) + rn and f(rn, n + 1) = f(n2, n) n

for all rn, n E N.


Find all pairs (pq) such that f(p,q) = 20th.

Problem 17
Let f be a function defined on [0, 1] such that

f(0) = 1(1) 1 and (f(a) f(b)I < a bI.


for all a b in the interval [0, 1].
Prove that
1
1(a) f(b)( <

Problem 18
Find all pairs of integers (x, y) such that
x3 + y3 = (x + y)2.

Problem 19
2
Let f(x) = for real numbers x.
4x + 2
Evaluate
1 1 \ / 2 \ /2000

Problem 20
Prove that for ii 6 the equation
1 1 1

x1 x2

has integer solutions.

Problem 21
Find all pairs of integers (a, b) such that the polynomial ax'7 + bx16 + I
is divisible by x2 x 1.
1. IntroductorY Problems

Problem 22
Given a positive integer n, let p(n) be the product of the non-zero digits
of n. (If n has only one digit, then p(n) is equal to that digit.) Let
+p(999).
What is the largest prime factor of S?

Problem 23
Let be a sequence of nonzero real numbers such that

xn =

forn = 3,4
Establish necessary and sufficient conditions on x1 and x2 for to be
an integer for infinitely many values of ri.

Problem 24
Solve the equation
3x =

Problem 25
For any sequence of real numbers A = {ai,a2,as,. }, define to be
the sequence {a2 a1, a3 a2, a4 a3.. .
. }. Suppose that all of the terms
of the sequence are 1, and that a1g = a92 = 0.
Find a1.

Problem 26
Find all real numbers x satisfying the equation
2x+3x 4X+6X9X =1.

Problem 27
Prove that

16 <17.

Problem 28
Determine the number of ordered pairs of integers (rn n) for which mn

m3 + n3 + 99mn =
6 1. Introductory Problems

Problem 29
Let a, b, and c be positive real numbers such that a + b + c 4 and
ab 4-bc+ca 4.
Prove that at least two of the inequalities

abl 2, lbcl 2, Ical

are true.

Problem 30
Evaluate
1

Problem 31
Let 0 < a < 1. Solve
= ax

for positive numbers x.

Problem 32
What is the coefficient of x2 when

is expanded?

Problem 33
Let n be distinct positive integers.
Find the maximum value of lxm where x is a real number in the
interval (0,1).

Problem 34
Prove that the polynomial

where ai. a2, .. are distinct integers, cannot be written as the prod-
,

uct of two non-constant polynomials with integer coefficients, i.e., it is


irreducible.
Problem 35
Find all ordered pairs of real numbers (x. y) for which:
(1+x)(1+x2)(1+x4)
and (1+y)(1+y2)(1+y4) = 1+x7.

Problem 36
Solve the equation
2(2x 1)x2 + (2z2 2)x = 2

for real numbers x.

Problem 37
Let a be an irrational number and let n be an integer greater than 1.
Prove that

is an irrational number.

Problem 38
Solve the system of equations
(x1 x2 -I-- x3)2 = x2(x4+x5x2)
(x2 x3 + x4)2= x3(x5+x1x3)
(x3 x4 + x5)2=
(x4 x5 + x1)2 = x5(x2+x3x5)
(x5 xi + = x1(x3+x4x1)
for real numbers Xl, x2, x4, x5.

Problem 39
Let x, y, and z be complex numbers such that

x y +z= 2,

+ + z2 = 3
and

STAATS.UL.NIV)
11
Xy+z1 + yz+x1 + zx+y1

= 4.
8 1. Introductory Problems

Problem 40
Mr. Fat is going to pick three non-zero real numbers and Mr. Taf is going
to arrange the three numbers as the coefficients of a quadratic equation

_x2 +_x+_=O.
Mr. Fat wins the game if and only if the resulting equation has two
distinct rational solutions.
Who has a winning strategy?

Problem 41
Given that the real numbers a, b, c, d, and e satisfy simultaneously the
relations
a+b+c+d+e=8anda2-j-b2+c2+d2+c2 =16,
determine the maximum and the minimum value of a.

Problem 42
Find the real zeros of the polynomial
Pa(S) = (x2 + 1)(x 1)2 ax2,

where a is a given real number.

Problem 43
Prove that
1 3 2n1 1

2 4 2n
for all positive integers n.

Problem 44
Let
P(x) =
a nonzero polynomial with integer coefficients such that P(r) =
P(s) = 0 for some integers r and s. with 0 < r < s.
Prove that ak < s for some k.

Problem 45
Let rn be a given real number.
Find all complex numbers x such that
(S \ (X\
+(
/
2
=m +772.
Problem 46
The sequence given by xo = a, .r1 = b, and
11 1

is periodic.
Prove that ab = 1.

Problem 47
Let a, b, c, and d be real numbers such that

(a2 +b2 i)(c2 +d2 1)> (ac+bd 1)2.

Prove that
a2 + b2> 1 and c2 + d2> 1.

Problem 48
Find all complex numbers z such that

(3z + 1)(4z + 1)(6z + 1)(12z + 1) = 2.

Problem 49
Let be the zeros different from 1 of the polynomial
P(x) = 1, n 2.
Prove that
1 1 ni
ix1
+
ix2
+...+ 1

Problem 50
Let a and b be given real numbers. Solve the system of equations

= a
V1_x2+y2

for real numbers x and y.


ADVANCED PROBLEMS
Z ADVANCED PROBLEMS

Problem 51
Evaluate
(2000'\ (2000\ (2000\ (2000

Problem 52
Let x, y, z be positive real numbers such that x4 + y4 -i- z4 = 1.

Determine with proof the minimum value of


x3 z3
1x 8+

ly8 1z8

Problem 53
Find all real solutions to the equation
2X + 3X + 6X = x2.

Problem 54
Let {an}ni be a sequence such that = 2 and
1
= +

for all n E N.
Find an explicit formula for

Problem 55
Let x, y, and z be positive real numbers. Prove that
2;
+

+
z+
z
y)
1.
14 2. Advanced Problems

Problem 56
Find, with proof, all nonzero polynomials f(z) such that
f(z2) + f(z)f(z +1) =0.

Problem 57
Let f N N be a function such that f(n + 1) > f(n) and
f(f(n)) = 3n

for all n.
Evaluate 1(2001).

Problem 58
Let F be the set of all polynomials f(x) with integers coefficients sueh
that f(x) = 1 has at least one integer root.
For each integer k > 1. find the least integer greater than 1 for
which there exists f E P such that the equation f(x) = rnk has exactly
k distinct integer roots.

Problem 59
Let x1 = 2 and
= 1,

for ii i.
Prove that
1 1 1 1 1
1
22 .r1 x2 22

Problem 60
Suppose that f : is a decreasing function such that for all
x,y E

f(x + y) + f(f(x) + 1(y)) = 1(1(1 + f(y)) + f(y + 1(x))).


Prove that f(f(x)) = x.
15

Problem 61
Find all functions f : Q
Q such that

f(x + y) + f(x y) = 2f(x) + 2f(y)

for all x,y EQ.

Problem 62
Let <a < 1.
Prove that the equation
x3(x + 1) = (x + a)(2x + a)

has four distinct real solutions and find these solutions in explicit form.

Problem 63
Let a, b, and c be positive real numbers such that abc = 1.

Prove that
1 I 1
1
+ <1.
a+b+1 b+c-I-1 c+a+1
Problem 64
Find all functions f, defined on the set of ordered pairs of positive inte-
gers, satisfying the following properties:

f(x, x) = x, f(x, y) = f(y, x). (x + y)f(x. y) yf(x. x + y).

Problem 65
Consider n complex numbers zk. such that IzkI 1, Ic = 2 n.
Prove that there exist c1. a2.... E fi. 1} such that, for any vi fl,

e1z1 + + + <2.

Problem 66
Find a triple of rational numbers (a. c) such that
16 2. Advanced

Problem 67
Find the minimum of

log11 (X2-_ + log12 +


where x1,x2,. .. are real numbers in the interval 1).

Problem 68
Determine x2 + + z2 + w2 if

22_12+ 22_32+22_52 =1,

42_12 + 42_32 +4252 =1,

62_12 +6232+6252+6272 =L

82_12 + 82_32 82_52 + 82_72 =1


-L

Problem 69
Find all functions f R : R such that
f(xf(x) + f(y)) = (f(x))2 +
for all x,y E

Problem 70
The numbers 1000, iOOl, ,2999 have been written on a board.
Each time, one is allowed to erase two numbers, say, a and b, and replace
them by the number min(a. b).
After 1999 such operations, one obtains exactly one number c on the
board. Prove that c < 1.
Problem 71
Let a1,a2,. . .be real numbers, not all zero.
Prove that the equation

has at most one nonzero real root.


Advanced Problems
2.

Problem '72
real numbers defined by a1 = t and
Let {an) be the sequence of
=

for n 1.
For bow many distinct values oft do we have a1998 = 0?

Problem '73

(a) Do there exist functions f : R and g R R such that


f(g(x)) = x2 and g(f(x)) =
for all x E R?
(b) Do there exist functions f: R and 9: R such that
f(g(x)) = x2 and g(f(x)) =
for all x E R?

Problem 74
LetO<a1 a2 < O<b1 <b2 berealnumberssuchthat

Suppose that there exists 1 k n such that for 1 i k and


b2 for i > k.
Prove that
a1a2 b1b2

Problem 75
Given eight non-zero real numbers a1, a2,. a8. prove that at least one
of the following six numbers: a1a3 + a2a4, a1a5 + a2a6, a1a7 + a2a8,
a3a5 + a4a6, a3a7 + a4a8. + a6a8 is non-negative.
Problem '76
Let a, b and c be positive real numbers such that abc = 1.
Prove that
ab bc ca
a5+b5+ab+b5+cS+bc+c5+a5+caL
18 2. Advanced Problems

Problem 77
Find all functions f: R * R such that the equality

f(f(x) + y) = f(x2 y) + 4f(x)y


holds for all pairs of real numbers (x. y).

Problem 78
Solve the system of equations:
3x y
=3
x2 + y2
x+3y
11 =0

Problem 79
Mr. Fat and Mr. Taf play a game with a polynomial of degree at least 4:

+... +_x + 1.
They fill in real numbers to empty spaces in turn. If the resulting poly-
nomial has no real root, Mr. Fat wins; otherwise, Mr. Taf wins.
If Mr. Fat goes first, who has a winning strategy?

Problem 80
Find all positive integers k for which the following statement is true: if
F(x) is a polynomial with integer coefficients satisfying the condition

for c=0.1 k+1,


then F(0)= F(1)= =F(k+ 1).
Problem 81
The Fibonacci sequence is given by

(nEN).
Prove that
2n+2 ' 2n2
bf2n
for all n 2.
ced ProblemS 19

Problem 82
R for which there exists a strictly monotonic
Find all functions u :
function f : R p R such that

f(x + y) = f(x)u(y) + f(y)

for all x, y c R.

Problem 83
Let Zn be complex numbers such that

IzlI+Iz2H+IzflI=1.
Prove that there exists a subset S of {z1, Z2,.. . . such that

Problem 84
A polynomial P(x) of degree n 5 with integer coefficients and n distinct
integer roots is given.
Find all integer roots of P(P(x)) given that 0 is a root of P(x).

Problem 85
Two real sequences x1, x2, . and Yi Y2. . are defined in the following
way:

and
yn
Yn+1
= 1 + yi +
for all n 1. Prove that 2 < <3 for all a > 1.
Problem 86
For a polynomial P(x), define the difference of P(x) on the interval [a. b]
-
([a,b), (a,b). (a.b]) as P(b) P(a).
Prove that it is possible to dissect the interval [0. 1[ into a finite number
of intervals and color them red and blue alternately such that. for every
polynomial P(x). the total difference of P(.r) on red intervals
is equal to that of P(x) on blue intervals.
What about cubic polynomials?
20 2. Advanced Problems

Problem 87
Given a cubic equation

x3 + _x2 + _x + = 0,
Mr. Fat and Mr. Taf are playing the following game. In one move, Mr.
Fat chooses a real number and Mr. Taf puts it in one of the empty spaces.
After three moves the game is over. Mr. Fat wins the game if the final
equation has three distinct integer roots.
Who has a winning strategy?

Problem 88
Let n > 2 be an integer and let f R2 R be a function such that for
any regular n-gon A1A2 . . .

f(A1) + f(A2) + . +

f is the zero function.


Problem 89
Let p be a prime number and let f(x) be a polynomial of degree d with
integer coefficients such that:
(i) f(0) = 0, f(l) = 1;

(ii) for every positive integer n, the remainder upon division of


by p is either 0 or 1.
Prove that d p 1.

Problem 90
Let n be a given positive integer.
Consider the sequence a0,a1, with a0 = and

ak = 4
TI

fork=l,2..n,
Prove that
1 _!
TI
< 1.
Advanced problems 21
2.

Problem 91
be nonnegative real numbers. not all zero.
Let ai,a2,."
(a) Prove that = 0 has precisely one
positive real root R.

(b) Let A = a2 and B= ja3.

Prove that AA <RB.

Problem 92
Prove that there exists a polynomial P(x, y) with real coefficients such
that P(x, y) 0 for all real numbers .r and y. which cannot be written
as the sum of squares of polynomials with real coefficients.

Problem 93
For each positive integer Ti. show that there exists a positive integer k
such that
/c = f(x)(x + 1)2n + + 1)
for some polynomials f.g with integer coefficients, and find the smallest
such Ic as a function of n.

Problem 94
Let x be a positive real number.
(a) Prove that
(ii 1)! 1

(x 1).. (x + n) I
.

(b) Prove that

(ni)! 1
22 2. Advanced

Problem 95
Let ii 3 be an integer, and let

Xcs={1.2 n3}

be a set of 3n2 elements.


Prove that one can find nine distinct numbers c7, (1 = 1. 2, 3) in X
such that the system
= 0

a2x+h2y+c2z = 0

a3x+b3y+c3z = 0

has a solution (XO, yo, zO) in nonzero integers.

Problem 96
Let n 3 be an integer and let Xi. i2. he positive real numbers.

Suppose that =
1

Prove that

Problem 97
Let x1,x2 be distinct real numbers. Define the polynomials

P(x) = (x x1)(.r ..

and
Q(x)=P(x)( (1 +
1
-+. +
\.iX1 XX2
Let y1,y2.. . . 'Ynl be the roots of Q. Show that
<
tJ
Advanced Problems
2.

Problem 98
Show that for any positive integer fl. the polynomial
f(x) = (x2 + +1

cannot be written as the product of two non-constant polynomials with


integer coefficients

Problem 99
Let 11,12.13 R R be functions such that

a1f1 + a2]'2 + a3f3

is
monotonic for all a1,a2,a3 R.

Prove that there exist c1, c2. C3 R. not all zero. such that

cifi(x) +c2f2(x) +c3f3(x) = 0

for all x Ift.

Problem 100
Let x1.x2, be variables, and let be the sums of
. .

nonempty subsets of
Let pk(xl be the elementary symmetric polynomial in
the yj (the sum of every product of A- distinct yjs).
For which k and n is every coefficient of Pk (as a polynomial in x1,. .. ,

even?
For example, if n = 2. then yi' Y2. y3 are Xl. + s2 and

Pi = + Y2 + = + 2x2,
P2 = Y1Y2 + 112Y3 + Y3Y1 = + +
P3 = Y1Y2Y3 = X1X2 +

Problem ioi.
Prove that there exist 10 distinct real numbers al,a2.. . aio such that
the equation

(xai)(x_a2)...(xaio) = (X+al)(X+a2) (x+aio)


has exactly 5 different real roots.
SOLUTiONS TO
iNTRODUCTORY PROBLEMS
3. SOLUTIONS TO
PROBLEMS

problem 1 [Romania 1974]


Let a, b, and c be real and positive parameters.
Solve the equation
\/c+ax= v'bax -f-s/c

Solution 1
It is easy to see that x = 0 is a solution. Since the right hand side is a
decreasing function of x and the left hand side is an increasing function
of x, there is at most one solution.
Thus x = 0 is the only solution to the equation.

Problem 2
Find the general term of the sequence defined by x0 = 3, = 4 and
_2

for all n C N.
Solution 2
We shall prove by induction that = n + 3. The claim is evident for
n=O,1.
Fork 1, if Xk_1 =k+2andxk=k+3, then
kxk =(k+2)2 k(k+3)=k+4,
as desired.
This completes the induction.
28 3. Solutions to Introductory Problems

Problem 3 [AHSME 1999]


Let x1. x2, , x7, be a sequence of integers such that
. .

(i) n;
(ii) X1 19;

(iii)

Determine the minimum and maximum possible values of

Solution 3
Let a, b, and c denote the number of is, is, and 2s in the sequence,
respectively. We need not consider the zeros. Then a, b, c are nonnegative
integers satisfying
a+b+2c= l9anda+b+4c=99.
It follows that a = 40c and b 593c, where 0 c 19 (since b 0),
so
19+6c.
When c = (a = 40,b = 59), the lower bound (19) is achieved.
0

When c = 19 (a = 21, b = 2), the upper bound (133) is achieved.

Problem 4 [AIME 1997]


The function f, defined by
ax + b
f(x)
cx+d'
where a, b, c, and d are nonzero real numbers, has the properties
1(19) = 19, f(97) = 97, and f(f(x)) =

for all values of x, except


Find the range of f.
Solution 4, Alternative 1
For all x, f(f(x)) = .r, i.e.,
(ax + b'\
a(
\cx+d) x
(ax+b\
cx + dj
to Introductory Problems 29
3

i.e. (a2 + bc)x + b(a + d)


X.
c(a+d)x+bc+d2
i.e.
c(a + d)x2 + (d2 a2)x b(a + d) = 0,

which implies that c(a + d) = 0. Since c 0, we must have a = d.


The conditions f(19) = 19 and f(97) = 97 lead to the equations
and 972c=297a+b.
Hence
(972 192)c = 2(97 19)a.

It follows that a = 58c, which in turn leads to b = 1843c. Therefore

f(x)= 58x.x58
1843 1521

which never has the value 58.


Thus the range of f is R {58}.
Solution 4, Alternative 2
The statement implies that f is its own inverse. The inverse may be
found by solving the equation
ay + b
cy + d
fory. Thisyields
dxb
cx + a
The nonzero numbers a. b, c, and d must therefore be proportional to d,
c, and a, respectively; it follows that a = d, and the rest is the
same as in the first solution.

Problem 5
Prove that
(ab)2 < a+b (ab)2
8a 2 8b
forallab> 0.

Alternative 1
that

) -
30 3. Solutions to Introductory Problems

i.e.

4a 4b

i.e.
(ab)2 < < (ab)2

from which the result follows.


Solution 5, Alternative 2
Note that
(a-t-b\2
-ab
(a-b)2
2 2(a + b) +
+

Thus the desired inequality is equivalent to

4a a+b+ 4b.

which is evident as a b> 0 (which implies a b).


Problem 6 [St. Petersburg 1989]
Several (at least two) nonzero numbers are written on a board. One may
erase any two numbers, say a and b, and then write the numbers a +
and b instead.
Prove that the set of numbers on the board. after any number of the
preceding operations, cannot coincide with the initial set.
Solution 6
Let S be the sum of the squares of the numbers on the board. Note that
S increases in the first operation and does not decrease in any successive
operation, as

a)2 =
(a+
with equality only if a = b = 0.
This completes the proof.
to Introductory Problems 31

problem 7 [AIME 1986]


The

may be written in the form


ao + aiy + a2y2 + . - + ai6y'6 + a17y17,

where y= x + 1 and are constants. Find a2.


SolutiOn 7, Alternative 1
Let f(x) denote the given expression. Then
xf(x)=xx2+x3 .x18

and
(1 + x)f(x) = 1

Hence
1 (y 1)18 1 (y 1)18
f(x)=f(y1)=
l+(yl) y
Therefore a2 is equal to the coefficient of y3 in the expansion of
1 (y 1)18,

i.e.,

a2 = 816.
=
Solution 7, Alternative 2
Let 1(x) denote the given expression. Then

f(x) =f(y1) l(y l)-f(y

Thus
/2\ /3\. (17\ (18"

Here we used the

I 7i '\
/n+1"

and the fact that


32 3. Solutions to Introductory Problems

Problem 8
Let a. b, and c be distinct nonzero real numbers such that
-1 = b+ -1 = c+ -.1
b c a
Prove that = 1,

Solution 8
From the given conditions it follows that
bc ca ab
ab=.bc,andca=.
bc ca ab
Multiplying the above equations gives (abc)2 = 1, from which the desired
result follows.

Problem 9 [Putnam 1999]


Find polynomials f(s), g(x), and h(s). if they exist. such that for all x
( 1 ifr<1
g(')I + h(s) = 3x + 2 if 1 < .r < 0
2x+2 ifx>0.
Solution 9, Alternative 1
Since x = 1 and x = 0 are the two critical values of the absolute
functions, one can suppose that
F(s) =
I (cab)x+da ifs <1
= if1<x<0
1(a+b+c)x+a+d ifx>0.
which implies that a 3/2, b = 5/2. c = 1. and d = 1/2.
Hence f(s) = (3s + 3)/2, g(x) = 5x/2, and h(s) = x +
Solution 9, Alternative 2
Note that if r(x) and 8(x) are any two functions, then

max(r,s)
r-l-s+lrsj
= 2

Therefore, if F(s) is the given function. we have


F(x) = max{3x 3. 0} max{5x, 0} + 3x + 2
= (3x3-'-j3x+31)/2(5x+15x1)/2-4-3x--i-2
=
colutions to Introductory Problems 33

Problem 10
Find all real numbers x for which
8X 7

12x + 18x 6

Solution 10
By setting = a and = b. the equation becomes
a3+b3 _7
a2b+ b2a 6

i.e.
a2ab+b2 _7
ab 6'
i.e.
Ga2 l3ab+6b2 = 0.

i.e.
(2a 3b)(3a 2b) = 0.

Therefore 2x+i = 3x+i or 2x1 = 3x1, which implies that x = 1 and


x = 1.
It is easy to check that both .r = 1 and x = 1 satisfy the given equation.

Problem 11 [Romania 1990]


Find the least positive integer in such that

(2n\ <ni
\n)
for all positive integers n.

ii
Note that

+
= (1+ i)2n =
< + ... +
and for n
(10)
= 252 >
Thus in
34 3. Solutions to Introductory Problems

Problem 12
Let a. b, c, d, and e be positive integers such that

abcde=a+b+c+d+e.
Find the maximum possible value of max{a, b, c. d, e}.
Solution 12, Alternative 1
Suppose that a b c d e. We need to find the maximum value of
e. Since
c<a+b+c+d+c<5c.
then e < abcdc 5e, i.e. < abcd 5. 1

Hence = (1.1,1,2), (1,1,1,3). (1. 1,1,4), (1, 1,2,2), or


(1,1, 1.5), which leads to max{e} = 5.

Solution 12, Alternative 2


As before, suppose that a b c d c. Note that
1 1 1 1 1

de d

d = 1, then a = b = c = 1 and 4 + c = e, which is impossible.


Thusd1 land ci e 5.

It is easy to see that (1, 1,1,2,5) is a solution.

Therefore max{e} = 5.

Comment: The second solution can be used to determine the maxi-


mum value of {x1, x2,. . when x1, x2, .
. , are positive integers . . .

such that
x1x2 . . = xi+ + + xm.

Problem 13
Evaluate

1!+2!+3!
3
+
2!+3!+4!
4
++ 2001
to Introductory Problems 35

13
Note that
k+2 k+2
= k![1+k+1+(k+1)(k+2)]
1

k!(lc+2)
k
(k+2)!
(k+2)1
(k-i--2)!

(k+1)! (k+2)!
By telescoping sum, the desired value is equal to

2 2001!

Problem 14

Find all possible values of x.


solution 14, Alternative 1
Since
\/a2 + al + 1> al
and
2a
Va2 +a+ 1+ Va2 -a+ 1'
we have
lxi < 12a/ai = 2,

Squaring both sides of

Va2+a+1
Yields

a + 1 = 2a x2.
36 3. Solutions to Introductory Problems

Squaring both sides of the above equation gives

x2(x24)
4(x
2
1)a =x (x2'4)ora =
2 2 2
4(21)'
Since a2 0, we must have
x2(x2 4)(x2 1) 0,
Since xI < 2, x2 4 < 0 which forces x2 1 < 0. Therefore, 1 < x < 1.
Conversely, for every x C (1, 1) there exists a real number a such that

x Va2 +a+1 a+1.

Solution 14, Alternative 2


Let A = B= and P = (a.O). Then P
is a point on the x-axis and we are looking for all possible values of
d=PA-PB.
By the Triangle Inequality, IPA PBJ < = 1. And it is clear
that all the values 1 <d < 1 are indeed obtainable. In fact, for such
a d, a half hyperbola of all points Q such that QA QB = d is well
defined, (Points A and B are foci of the hyperbola.)
Since line AB is parallel to the x-axis, this half hyperbola intersects the
x- axis, i.e., P is well defined.

Problem 15
Find all real numbers x for which
lOx + lix + 12x = 13X + 14x.

Solution 15
It is easy to check that x = 2 is a solution. We claim that it is the only
one. In fact, dividing by 13X on both sides gives
112\x

The left hand side is a decreasing function of x and the right hand side
is an increasing function of x.
Therefore their graphs can have at most one point of intersection.
Solutions to Introductory Problems 37

Comment: More generally.


a2+(a+1)2+.+(a+k)2
=(a+k+1)2+(a+k+2)2+"+(a+2k)2
forak(2k+'). kcN.
problem 16 [Korean Mathematics Competition 2001]
Let f: N x N N be a function such that f(1, 1) = 2.

f(m + 1. n) = f(ni, n) + ni and f(772, 72 + 1) f(n2. n)

for all rn, n c N.


Find all pairs (p, q) such that f(p. q) = 2001.

Solution 16
We have

f(pq) = f(pl,q)+p1
=

=
2

q(q1) p(pl)
= f(1,l)
2 2
= 2001.

Therefore
p(p 1) q(q 1)
1999,
2 2
i.e.

(pq)(p+q 1) =2. 1999.


ROte that 1999 is a prime number and thatpq < p+q 1 for p.q c N
We have the following two cases:

1 = 1999. Hence p= 1001 and q= 999.


38 3. Solutions to Introductory Problems

Therefore (p. q) = (2000, 1999) or (1001,999).

Problem 17 [China 1983]


Let f be a function defined on [0, 1] such that

f(0) = f(1) = land f(a)f(b)I < labi,


for all a bin the interval [0, 1].
Prove that
f(a) f(b)j <

Solution 1.7
We consider the following cases.

1. a bI < 1/2. Then If(a) f(b)f < a - bI as desired.

2. Ia bi > 1/2. By symmetry, we may assume that a> b. Then

f(a) f(b)I = If(a) 1(1) + f(0) f(b)I


< f(a) f(')I + 1(0) f(b)I
< IalI+I0bI
= 1a+b0
= 1(a-b)

as desired.

Problem 18
Find all pairs of integers (x, y) such that
+ = (x + y)2.

Solution 18
Since x3+y3 = (x+y)(x2xy+y2), all pairs of integers (nri), n C 7Z.
are solutions.
Suppose that x + y 0. Then the equation becomes
S2 Xy + y2 = x + y,

i.e.
solutions to Introductory Problems 39
3

a quadratic equation in x, we calculate the discriminant

A = y2 + 2y + 1 4y2 + 4y = 3y2 + 6y + 1.

solving for A 0 yields

3 3

Thus the possible values for y are 0, 1, and 2. which lead to the solutions
(1,0), (0,1), (1,2), (2.1), and (2,2).
Therefore, the integer solutions of the equation are (x. y) = (1, 0), (0, 1),
(1,2), (2,1), (2,2), and (n, n). for all nEZ.

Problem 19 [Korean Mathematics Competition 2001]


Let
2
f(x)= 4X+2
for real numbers x. Evaluate
/ 1 \ / 2 '\ /2000

f has a half-turn symmetry about point (1/2, 1/2). Indeed,


2 2.4x 4X

from which it follows that 1(x) -t- f(1 x) 1.


Thus the desired sum is equal to 1000.

Problem 20
Prove that for n 6 the equation

1 1 1

x1
has integer solutions.
Solution 20
Note that
1 1 1 1 1
40 3. Solutions to Introductory Problems

from which it follows that if (x1, x2, , = (a1 .a2, is an inte.


ger solution to
1 1 1

x1 x2

then

(x1,x2,. .

is an integer solution to
1 1 1

xl x2

Therefore we can construct the solutions inductively if there are solutions


for n = 6, 7, and 8.
Since xi = 1 is a solution for n = 1, (2,2,2,2) is a solution for n = 4,
and (2, 2, 2,4,4.4,4) is a solution for n = 7.
It is easy to check that (2, 2, 2,3, 3, 6) and (2, 2. 2, 3,4,4, 12, 12) are solu-
tions for n = 6 and n = 8, respectively. This completes the proof.

Problem 21 [AIME 1988]


Find all pairs of integers (a, b) such that the polynomial

ax17 + bx'6 -i- 1

is divisible by x2 x 1.

Solution 21, Alternative 1


Let p and q be the roots of x2 x I = 0. By Vieta's theorem,
p + q = 1 and pq 1. Note that p and q must also be the roots of
ax17 + bx'6 + 1 = 0. Thus

ap17 + bp'6 = 1 and aq17 + bq'6 1.

Multiplying the first of these equations by q'6, the second one by p16,
and using the fact that pq = 1, we find

ap+b=q16andaq+b=p'6. (1)

Thus
q'6
a=
I = (p8 + q8)(p4 + q4)(p2 + q2)(p q).
solutions to Introductory Problems 41

since
p+q = 1,
p2+q2 = (p+q)22pq=1+2=3,
p4+q4 = (p2+q2)22p2q2=92=7,
p8+q8 = (p4+q4)22p4q4=49247,
follows that a 1 3 7 . 47 = 987.
Likewise, eliminating a in (1) gives

b = p'7q'7
pq
= p'6+p'5q+p'4q2++q16
= (p'6 + q16) + pq(p'4 + q14) + p2q2 (p'2 + q'2)
+ p7q7(p2 + q2 ) + p8q8
= (p16 +q'6) (p'4 +q'4)+ (p2 +q2)+ 1.
For n 1, let k2 = 3 and k4 = 7, and
1. 2n+4 2n+4
2n+4 P
= (p2fl+2
+ + q2) + q2Th)
=

for n 3. Then k6 18, k8 = 47, k1,, = 123, k,2 = 322, k,4 843,
k16 = 2207.
Hence

b=2207843+322123+47 18+73+1=1597
or
(a, b) = (987, 1597).

solution 21, Alternative 2


The other factor is of degree 15 and we write
(C15x15 C14X14 + .. + cix x 1) = ax17 + bx'6 + 1.
Comparing coefficients:

x0: c0=1.
c0c,=0,c,=1
x2: coc,+c2=0.c2=2,
andfor3<k<15, xk: Ck_2Ck_,+CkO.
42 3. Solutions to Introductory

It follows that for k 15, = Fk+1 (the Fibonacci number).


Thus a = = F16 = 987 and b c14 c15 = F17 = 1597
(a, b) = (987, 1597).

Comment: Combining the two methods, we obtain some interesting


facts about sequences and Since

= = =
it follows that and satisfy the same recursive relation. It ,i5
easy to check that k2 = F1 + F3 and k4 = F3 + F5.
Therefore = + and

2n+1
I.2m 2n2 -rLb2n4 Lf 2
L1

Problem 22 [AIME 1994]


Given a positive integer n, let p(n) be the product of the non-zero digits
of n. (If n has only one digit, then p(n) is equal to that digit.) Let
S=p(l)+p(2)+ +p(999).
What is the largest prime factor of 8?
Solution 22
Consider eachpositive integer less than 1000 to be a three-digit number
by prefixing Os to numbers with fewer than three digits. The sum of the
products of the digits of all such positive numbers is

(0O.O+OO.1+...+9.99)O.O.O

However, p(n) is the product of non-zero digits of n. The sum of these


products can be found by replacing 0 by 1 in the above expression, since
ignoring 0's is equivalent to thinking of them as l's in the products. (Note
that the final 0 in the above expression becomes a 1 and compensates
for the contribution of 000 after it is changed to 111.)
Hence

S 1 = (46 1)(462+46+ 1) = 33 5 7.103,

and the largest prime factor is 103.


Solutions to Introductory Problems 43

problem 23 [Putnam 1979]


of nonzero real numbers such that
Zn be a sequence
Xn_2Xm_1
xn '

for n = 3,4
establish necessary and sufficient conditions on x1 and x2 for to be
an integer for infinitely many values of n.
Solution 23, Alternative 1
We have
1 2

Xn_2Xn_1

Let = Then = Yn1 Yn2. i.e.. y,-, is an arithmetic


sequence. If is a nonzero integer when n is in an infinite set 8, the
yb's for n C S satisfy 1 1.
Since an arithmetic sequence is unbounded unless the common difference
is 0, 7/n Yni = 0 for all n, which in turn implies that x1 = = m, a
nonzero integer.
Clearly, this condition is also sufficient.
Solution 23, Alternative 2
An easy induction shows that
x1x2 x1x2
xn_
(n 1)xi (n 2)x2 (x1 x2)n + (2x2 x1)

for n 3,4
In this form we see that will be an integer for infinitely many values
of n if and only if x1 = = in for some nonzero integer m.
Problem 24
Solve the equation
3x =
Solution 24, Alternative 1
It is clear that 2. We consider the following cases.
1. 2 x < 2. Setting x = 2cosa, 0 < a < ir, the equation becomes
8cos3 a 6cosa = + 1).
or
44 3. Solutions to Introductory Problems

from which it follows that cos 3a = cos

Then 3a = 2m7r, rn or 3a + = 2n7r, n E Z.


Since 0 a the solution in this case is
4ir 4ir
x=2cos0=2, andx=2cosT.
2. x> 2. Then x3 4x = x(x2 4) > 0 and

x2x2=(x2)(x4-1) >0
or
x>
It follows that
3x > x>
Hence there are no solutions in this case.
Therefore, x = 2, x = 2cos4ir/5, and x = 2cos4ir/7.
Solution 24, Alternative 2
For x> 2. there is a real number t > 1 such that

x=t 21
The equation becomes

(t7 1)(t5 1) = 0,

which has no solutions for t> 1.


Hence there are no solutions for x > 2.
For 2 x 2, please see the first solution.
Solutions to Introductory Problems 45

problem 25 [AIME 1992]


any
sequence of real numbers A = {ai, a2, a3, }, define AA to be
For

Suppose that all of the terms of the sequence are 1, and that
= a92 0.

Find al
SolutiOn 25
Suppose that the first term of the sequence is d.
Then
AA={d,d+1,d+2,..}
with the term given by d + (n 1).

Hence

with the term given by

=a1 + (n 1)d+ 1)(n 2).

This shows that a quadratic polynomial in ii with leading coefficient


1/2,
Since a19 = a92 = 0. we must have

= 19)(n 92),

soa1 =(119)(1 92)/2=819.

Problem 26 [Korean Mathematics Competition 2000]


Find all real numbers x satisfying the equation
2X + 3X 4X + 6X 9X =

solution 26
Setting = a and = b, the equation becomes

1-i-a2 -i-b2 a bab= 0.


Multiplying
both sides of the last equation by 2 and completing the
Square5 gives
(1a)2+(ab)2+(b- 1)2
46 3. Solutions to Introductory Problem

Therefore 1 = = 3X and x = 0 is the only solution.

Problem 27 [China 1992]


Prove that
16< <17.

Solution 27
Note that

Therefore

which proves the lower bound.


On the other hand.

Therefore

17.

which proves the upper bound. Our proof is complete.

Problem 28 [AHSME 1999]


Determine the number of ordered pairs of integers (m. m) for which mm
O and
in3 + n3 + 99mn =

Solution 28
Note that (m + n)3 = m3 + n3 + 3mn(m + n). If m + n = 33. then
333
= (in + n)3 = m3 n3 + 3nin(m + n) = in3 + + 99mm.

Hence m + n 33 is a factor of in3 + n3 + 99inn We have

+ n3 + 99mm 333

= n 33)[(m m)2 + (m + 33)2 + (n + 33)2]


5olution5 to Introductory Problems 47

Uence
there are 35 solutions altogether: (0,33), (1.32), (33,0), and
(33, -33).
More generally, we have
corn ient:
a3 + + c3 3abc

problem 29 [Korean Mathematics Competition 2001]


Let a, b, and c be positive real numbers such that a + b c 4 and
ab+bc+ca 4.
Prove that at least two of the inequalities

abI2,
are true.
Solution 29
We have
(a+b+c)2 <16.
i.e.
a2+b2+c2+2(ab+bc+ca)<16,
i.e.
a2 + b2 -f c2 8,
i.e.
a2+b2+c2(ab+bc+ca) 4,
i.e.
(ab)24-(bc)2+(ca)2 8,
and the desired result follows.

Problem 30
Evaluate
1
48 3. Solutions to Introductory

Solution 30
Let denote the desired sum. Then

1 (2n)!
s
(2n)!k(n_k)!(n+k)!
n
1 ,' 2n

/
1 (272

(2n)! 2

(2n)! 2(n!)

Problem 31 [Romania 1983]


Let 0 < a < 1. Solve
= aX

for positive numbers x.


Solution 31
Taking loge yields
aX loge x =
Consider functions from
f(x)=aX. g(x)=Iogax, h(x)=xa.

Then both f and are decreasing and h is increasing. It follows that


f(x)g(x) = h(x) has unique solution x = a.

Problem 32
What is the coefficient of x2 when

is expanded?
to Introductory Problems 49
3

SolUtbohl 32
Let
= an.o + afl1X+
is eaSY to see that = 1 and
It

Since
=
= (1 + 1)x + +...) (1 + 2Thx)
=
we have
+

= a1,2 + + 26 + + 22n) (22 + + +

=2+ 24(2272_2 1) - 3
1)

22Th+2_3.2n+1 +2 (2n+1 1) (2m+1 2)


3 3

Problem 33
Let in and n be distinct positive integers.
Find the maximum value of x is a real number in the
interval (0,
Solution 33
By symmetry, we can assume that rn > n. Let y =
Since0 < x < 1, < xTh and 0 < y <1. Thus
== Xmn) =

I

APplying the AM-GM inequality yields

ri _y)m_fl
= (
72
/

(n. +(m n)(1
n+mn

=
mm
50 3. Solutions to Introductory

Therefore

(
Is x It \ mm J
I =(mn)(
Equality holds if and only if

(rnn)y
1

or
In \
\m

Comment: For m = n + 1, we have


5n+1 <

for real numbers 0 <x <1. Equality holds if and only if x = n/(n + 1).

Problem 34
Prove that the polynomial

where a1, a2, , are distinct integers, cannot be written as the prod-
uct of two non-constant polynomials with integer coefficients, i.e., it is
irreducible.
Solution 34
For the sake of contradiction, suppose that

is not irreducible. Let f(s) = p(x)q(x) such that p(x) and q(x) are two
polynomials with integral coefficients having degree less than n. Then

g(x) = p(x) + q(x)


is a polynomial with integral coefficients having degree less than n.
Since
p(aj)q(aj) = f(aj) = 1
and both p(aj) and q(aj) are integers,

= = 1
to Introductory Problems 51

and
p(aj) + q(aj) = 0.

Thus
g(x) has at least ri roots. But degg <n, so g(x) = 0. Then

p(x) = q(x) and f(x) = p(x)2,

which implies that the leading coefficient of 1(x) must be a negative


integer, which is impossible, since the leading coefficient of f(x) is 1.

problem 35
Find all ordered pairs of real numbers (x, y) for which:

(1+x)(1+x2)(1+x4) = 1+y7
and (1+y)(1+y2)(1+y4) 1-f-x7.

Solution 35
We consider the following cases.
1. xy = 0. Then it is clear that x = y = 0 and (x, y) = (0,0) is a
solution.
2. xy <0. By the symmetry, we can assume that x >
0 > y. Then
(1+x)(1+x2)(1+x4) >1 and 1+y7 <1. Therearenosolutions
in this case.

3. x, y > 0 and x y. By the symmetry, we can assume that x >


y > 0. Then

(1 +x)(1 +x2)(1 +x4) >1 +x7>1 +y7,

showing that there are no solutions in this case.


4. x,y <0 and x y. By the symmetry, we can assume that x < y <
0. Multiplying by 1 x and 1 y the first and the second equation,
respectively, the system now reads

1x8 = (1+y7)(1x)=1x+y7xy7
1y8 = (1+x7)(1y)=1y+x7x7y.
Subtracting the first equation from the second yields

xS_yS=(x_y)-f-(x?_y7)_xy(x6_y6). (1)
Since x <y <0, xs > 0, x y < 0, x7 y7 <0, xy <0, and
X > 0.
Therefore, the left-hand side of (1) is positive while
the right-hand side of (1) is negative.

Thus there are no solutions in this case.


52 3. Solutions to Introductory Problems

5. x= y. Then solving

1 1 x xy7 1 x + X7 X8

leads to x = 0, 1, 1, which implies that (x, y) = (0.0) or (1, 1).

Therefore, (x, y) = (0,0) and (1, 1) are the only solutions to the
system.

Problem 36
Solve the equation

1)x2 + (2x2 2)x = 2x+1 2

for real numbers x.


Solution 36
Rearranging terms by powers of 2 yields

1)2(x2+x1) =0. (1)

Setting y = 1 and dividing by 2 on the both sides, (1) becomes


2Yx+2xy_(x+y) =0
or
(2)

Since f(x) = 1 and x always have the same sign,

0.
Hence if the terms on the left-hand side of (2) are nonzero, they must
have the same sign, which in turn implies that their sum is not equal to
0.
Therefore (2) is true if and only if x = 0 or y = 0. which leads to solutions
x = 1,0, and 1.

Problem 37
Let a be an irrational number and let n be an integer greater than 1.
Prove that
(a+ Va2_ + (a- -
is an irrational number.
to Introductory Problems 53
3

SolUtbol1 37
Let .1.

N= (a+ + (a

and let
1)*,

Then N = b + 1/b. For the sake of contradiction, assume that N is


rational. Then by using the identity

+ = + + - +
1)
repeatedly for m 1, 2 we obtain that btm + 1/btm is rational for all
m E N.
In particular,

=a+ 1+a- Va2 - 1=2a


is rational, in contradiction with the hypothesis.
Therefore our assumption is wrong and N is irrational.

Problem 38
Solve the system of equations

(x1 X2 x3)2 = x2(x4 + X5 x2)


(x2x3+x4)2 = x3(x5+x1x3)
(x3x4+x5)2 = x4(x1+x2x4)
= x5(x2+x3x5)
(x5x1-f-x2)2 = xi(x3+x4x1)
for real numbers x1, x2, x3, x4, S5.
38
Let 5k+5 = Adding the five equations gives
5 5

+ 2XkXk+2) = + 2XkXk+2).
k=1
It follow5 that

= 0.
54 3. Solutions to Introductory Problems

Multiplying both sides by 2 and completing the squares yields

Xk+1)2 = 0,

from which x1 = x2 = X3 = x4 = Therefore the solutions to the


system are
(xi,x2.x3,x4,xs) = (a, a, a. a, a)

for a C R.

Problem 39
Let x. y, and z be complex numbers such that x+y+z = 2, x2+y2+z2
3, and xyz = 4.
Evaluate
1 1 1
+ +
xy+z1 yz+xl zx+yl
Solution 39
Let S be the desired value. Note that

xy+zl =xy+1x--y=(xl)(y1).
Likewise,
yz+xl (y 1)(x 1)
and
zx+y 1 = (z 1)(x 1).
Hence
1 1 1
S = (x_1)(y_1)+(y_1)(z_1)+(z_1)(x_1)
x+y+z3 1
- (x-l)(y-l)(z--l)
1
xyz(xy+yz+zx)+x+y+z-- 1

1
5(xy+yz+zx)
But
2(xy + yz + zx) = (x + y + z)2 (x2 + + z2) = 1.

Therefore S = 2/9.
5oiutions to Introductory Problems 55

problem 40 [USSR 1990]


Mr Fat is going to pick three non-zero real numbers and Mr. Taf is going
8rrange the three numbers as the coefficients of a quadratic equation

x2+ x+ =0.
Mr. Fat wins the game if and only if the resulting equation has two
distinct rational solutions.
Who has a winning strategy?
Solution 40
Mr. Fat has the winning strategy. A set of three distinct rational nonzero
numbers a, b, and c, such that a + b + c = 0, will do the trick. Let A, B,
and C be any arrangement of a, b, and c, and let f(x) = Ax2 + Bx + C.
Then
f(1) = A + B + C = a+b + c =0.
which implies that 1 is a solution.
Since the product of the two solutions is C/A, the other solution is C/A.
and it is different from 1.

Problem 41 [USAMO 1978]


Given that the real numbers a, b, c, d, and e satisfy simultaneously the
relations

a+b+c+d+e=Sanda2+b2+c2+d2+c2= 16,
determine the maximum and the minimum value of a.
Solution 41, Alternative 1
Since the total of b.c. d. and c is S a, their average is x = (S a)/4.
Let
b=x+b1, c=x+c1, d=x+d1, e=x+c1.
Then b1 + d1 + c1 = 0 and
(8a)2
(1)
or
0 5a2 16a = a(5a 16).
Therefore 0 a 16/5, where a = 0 if and only if b = c = d = c = 2
nd a 16/5 if and only if cd 6/5
56 3. Solutions to Introductory Problem5

Solution 41, Alternative 2


By the EtMS-AM inequality, (1) follows from

and the rest of the solution is the same.

Problem 42
Find the real zeros of the polynomial
Pa(X) = (x2 + 1)(x 1)2 ax2.

where a is a given real number.


Solution 42
We have
(x2 + 1)(x2 2x + 1) ax2 0.

Dividing by x2 yields
I 1\I 1'\
I

xj\
x + I (x 2 + I a
xj 0.

By setting y = x + 1/x, the last equation becomes

y2 a = 0.

It follows that
x+- = 1
x
which in turn implies that, if a 0, then the polynomial Pa(X') has the
real zeros
1+
2
In addition, if a 8, then Pa(X) also has the real zeros

1- fiPi -
2

Problem 43
Prove that
1 3 2n1 1

2 4 2n
for all positive integers n.
Solutions to Introductory Problems 57

We use induction
For n = 1, the result is evident.
SuPPOSe the statement is true for some positive integer k, i.e.,
1 3 2ki 1

2k

Then
3 2k1 2k+1 1 2k+1
2k
In order for the induction step to pass it suffices to prove that
1 2k+1 I
2k+2 < y3k+4
This reduces to
(2k+1\2 3k+1
<3k+4'
i.e.
(4k2+4k+1)(3k+4) <(4k2+8k+4)(3k+1),
i.e.
0< k,
which is evident. Our proof is complete.

Comment: By using Stirling numbers, the upper bound can be im-


proved to for sufficiently large n.

Problem 44 [USAMO Proposal, Gerald Heuer]


Let
P(x) =
a nonzero
polynomial with integer coefficients such that

P(r) = P(.s) = 0

for Some integers


r and s, with 0 <r <s.
that ak s for some k.
58 3. Solutions to Introductory

Solution 44
Write P(x) = (x S)XCQ(X) and

Q(x) = b0xm + + ... +


where bm 0. Since Q has a positive root, by Descartes' rule of signs,
either there must exist some k for which bk > 0 bk4l, or bm > 0.
If there exists a k for which bk > 0 bk+1, then

ak+1 = sbk + bk+1 S.


If bm > 0, then = sbm S.

In either case, there is a k such that ak s, as desired.

Problem 45
Let rn be a given real number, Find all complex numbers x such that

IX
/
I1
'.2
\ IX
/ '.2
\
\x+1J \X1J +712.
+I1 =772 2

Solution 45
Completing the square gives

/ x \2
x 2x2
\,x+l xlj = x 21 +m2+rn,
i.e.
/ 2x2
\2 2x2 2
I = +712 +772.
x21
Setting y = 2x2/(x2 1), the above equation becomes

y (rn2 + m) = 0,
i.e.
(ym1)(y+m)=0.
Thus
2x2 2x2
=772or =712+1,
x21 x21
which leads to solutions

-2andx 1.
solutions to Introductory Problems 59

problem 46
The sequence given by xo = a, = b, and

= +
2\

js periodic.
prove that ab = 1.

solution 46
by on both sides of the given recursive relation yields
= +1

or
1) = 1.

Let = 1 for n C N. Since Yn+i = is a geometric


sequence. xn is periodic, then so is
If which implies that = 0 for
all n E N. Therefore
ab = XjXl = Yi + 1= 1.

Problem 47
Let a, b, c, and d be real numbers such that

(a2 + b2 1)(c2 + d2 1) > (ac+ bd 1)2.

Prove that
a2 + b2> I and c2 + d2 > 1.

Solution 47
For the sake of the contradiction, suppose that one of a2 + b2 or c2 + d2
'S less than or equal to 1. Since (ac+bd 1)2 0, a2 +b2 land
C2 + d2 1
must have the same sign. Thus both a2 b2 and c2 + d2 are
less than 1. Let

x=1a2b2andy=1c2---d2.
Thefl 0 < x, y 1. Multiplying by 4 on both sides of the given inequality

4xy > + 2bd 2)2 = (2 2ac 2bd)2


(2ac
= (a2+b2+x-i-c2+d2+y2ac2bd)2
= [(ac)2
(x+y)2=x2+2xy+y2,
60 3. Solutions to Introductory Problems

or 0> x2 2xy f y2 = (x y)2, which is irhpossible.


Thus our assumption is wrong and both a2 + b2 and c2 + d2 are
than 1.

Problem 48
Find all complex numbers 2 such that

(3z+1)(4z+ 1)(6z+ 1)(12z+ 1) =2.

Solution 48
Note that

8(3z + 1)6(4z + 1)4(6z 4- l)2(12z -1- 1) = 768,

i.e.
(24z+8)(24z +6)(24z +4)(24z+2) = 768.
Setting u = 24z + 5 and w = u2 yields

(u+3)(u+ 1)(u 1)(u3).= 768.

i.e.

(u2 1)(u2 9) =
i.e.
lOw 759 = 0,

i.e.

(w33)(w+23) =0.
Therefore the solutions to the given equation are

Z andz=
24 24

Problem 49
Let x1,x2, be the zeros different from 1 of the polynomial
P(x) = 1, n 2.
Prove that

1 1 1 ni
+ 1-
1x1 1x2 2
to Introductory Problems 61

5olutiofl Alternative 1

Q(x) = P(1 x) = (1

Then

Q(x) = (_1)nxnl + + +
-
and are the nonzero roots of the polynomial Q(x). as
(1ai =0.
Thus the desired sum is the sum of .the reciprocals of the roots of poly-
nomial Q(x), that is,
1 1 1
+
1 X1 I I

=++4-
a2
1

a1
1 1

= a2a3 + ala3 -- + a1a2

By the Vieta's Theorem, the ratio between

and

is equal to the additive inverse of the ratio between the coefficient of x


and the constant term in Q(x), i.e., the desired value is equal to
(n)

as desired.
Solution 49, Alternative 2
For any polynomial R(x) of degree ni, whose zeros are Xi. Xn_ 1'
the following identity holds:

1 1 R'(x)
XX1
+ +...+ 1
=
R(x)
62 3. So'utions to Introductory Problems

For
Xl 1 2
+...+x+1,
x1
R(1) = n and
n(n 1)
R'(i)=(n 1)+(n2)+'+ =
It follows that
1 1 1 R'(l) ni
1x1
+
1x2
+...+ R(1) 2

Problem 50
Let a and b be given real numbers.
Solve the system of equations

s/f x2 + y2
for real numbers x and y.
Solution 50
Let u = x,+ y and v = x y. Then
2 2 U+V UV

Adding the two equations and subtracting the two equations in the orig-
inal system yields the new system
= (a+b)s/1uv
=
Multiplying the above two equations yields
uv(1 uv) = (a2 b2)(1 uv).
hence uv = a2 b2. It follows that

u= (a+b)i/1a2+b2 andv= (ab)V1a2+b2


which in turn implies that
(a+bV'a2_b2 b+a'1a2b2
-i-b2

whenever 0 < a2 b2 < 1.


SOLUTiONS TO
ADVANCED PROBLEMS
4. SOLUTIONS TO
ADVANCED PROBLEMS

problem 51

f2000\ f2000\ 1 2000\ (2000

Solution 51
Let 2000
1(x) = (1 + x)200 =

Let w = (1 + Then w3 = 1 and w + 1 = 0. Hence


((2000\ (2000\ (2000

= f(1) + wf(w) + w2f(w2)


= 22000 + + + + w2)200
= 22000 + w(w2)200 + w2(w)200

= 22000 + + = 22000 1.

Thus the desired value is


22000 1

Problem 52
Let x, y, z be positive real numbers such that x4 + y4 + z4 = 1

Determine with proof the minimum value of

+ +
1 x8 1 y8 1 z8

52
For 0 < u < 1, let 1(u) = u(1 u8). Let A be a positive real number.
RY the AM-GM inequality,

[Au8 +8(1 _u8)]9


A(f(u))8 =Au8(1 _u8)..(1 _u8) <
66 4. Solutions to Advanced

Setting A = 8 in the above inequality yields

8(f(u))8 < (8)

or

It follows that

+ + + +
1 x8 1 y8 1 = x(1 x8) y(1 y8) z(1 z8)

>
8

with equality if and only if


1
x=y = z =

Comment: This is a simple application of the result of problem 33 in


the previous chapter.

Problem 53 [Romania 1990]


Find all real solutions to the equation
2X + 3X + 6X = x2.

Solution 53
Forx < 0, the function f(x) = + + x2 is increasing, so the
equation 1(x) = 0 has the unique solution x = 1.
Assume that there is a solution s 0. Then
3S + 6S
+ 3,

so s and hence 1.
But then s LsJ yields

2S 2LsJ = (1 + I+ s,
which in turn implies that
GS > 4S = (23)2
_____________

Solutions to Advanced Problems 67

28 + > s2, a contradiction.


x 1 is the only solution to the equation.

problem 54
j4et {an}n1 be a sequence such that al = 2 and
1
= +

for all n E N.
Find an explicit formula for
Solution 54
Solving the equation
x 1

2 x
leads to x Note that

Therefore,


2+1)

and
+i]
1

Problem 55
Let x, y, and z be positive real numbers. Prove that
x y
+
y+f(y+z)(y+x)
1.

55
Note that
.J(x+y)(x+z)
68 4. Solutions to Advanced Problems

In fact, squaring both sides of the above inequality yields

x2+yz
which is evident by the AM-GM inequality. Thus
x x

-
Likewise,
Y v'V
-
and
z <
-
Adding the last three inequalities leads to the desired result.

Problem 56
Find, with proof, all nonzero polynomials f(z) such that

f(z2) + f(z)f(z + 1) = 0.

Solution 56
Let 1(z) = azm(z where m and n are non-negative integers
and
g(z) = (z zi)(z Zk),

0 and ; 1, for i = 1,2,.. . , k. The given condition becomes

az2m(z + zi)(z2 (z2 Zk)


= 1)m(z z2)...(z zk)
.(z+1z1)(z+ 1z2)(z+1zk).
Thus a= a2, and f is nonzero, so a = 1. Since z2 1, 1 z2 0.
Then 22m = that is, m = n.
Thus f is of the form
zm(z 1)mg(z).

Dividing by Z2m(2 + the last equation becomes

g(z2) = g(z)g(z + 1).


,lutions to Advanced Problems 69

We claim that g(z) 1. Suppose not; then clearly g must have at least
one complex root r 0. Now

g(r2) = g(r)g(r + 1) = 0,
g(r4) = 0,
g(r8) = 0.

and so on.
Since g cannot have infinitely many roots, all its roots must have absolute
value 1.
Now,
g((r 1)2) = g(r 1)g(r) = 0,

soj(r1)21 = 1.

Clearly, if

then

2 ' 2

But r2 is also a root of g, so the same should be true of r2:

7'
2 fi+vi
2 2

This is absurd. Hence, g cannot have any roots, and g(z)


Therefore, the f(z) are all the polynomials of the form for
mEN,

Problem 57
Letf: N * N be a function such that f(n+ 1) > f(n') and f(f(n)) = 3n
for all ri.
Evaluate f(2001).
57, Alternative 1
We prove the following lemma.
Lemma For ri=0,1,2,...,
:1. and
2.
70 4. Solutions to Advanced Problems

Proof We use induction.


For n = 0, note that f(1) 1, otherwise 3 = f(f(1)) = 1(1) = 1, which
is impossible. Since f N N, f(1) > 1. Since f(n + 1) > 1(n),
f is increasing. Thus 1 < 1(1) < f(f(1)) = 3 or 1(1) = 2. Hence
I (2) = 1 (1 (1)) = 3.
Suppose that for some positive integer ri 1,

f
and
f (f(3n+1)) = 3n+2,

as desired. This completes the induction. 0


There are 1 integers m such that < in < 2
.
and there are
3fl 1 integers m' such that

f
3fl
2 +in,
for 0 m 3n Therefore

+m) =f(f (3fl +m)) =


Hence
36 (36
1(2001) = 1(2 .
+ 543) = 3 + 543) = 3816.

Solution 57, Alternative 2


For integer n, let fl(3) = a1a2 denote the base 3 representation of

Using similar inductions as in the first solution, we can prove that

I ifa1 =1.
= 1a2 . if al = 2.

Since 2001(3) = 2202010, f(2001)(3) = 12020100 or


37
1 (2001) = 1 32 +2 34
+2 36
+ = 3816.
Advanced Problems 71

problem 58 [China 1999]


Let F be the set of all polynomials f(x) with integers coefficients such
f(x) = 1 has at least one integer root.
For each integer k> 1, find the least integer greater than 1 for which
there exists an f E F such that 1(x) = mk has exactly k distinct integer
roots.
Solution 58
Suppose that fk E F satisfies the condition that 1k (x) = has exactly
k distinct integer roots, and let a be an integer such that fk(a) = 1. Let
be the polynomial in F such that

gk(X) fk(x+a)

for all x.
Now = fk(a) = 1, so the constant term of is 1. Now gk(x) = mk
has exactly k distinct integer roots r1, r2 rk, so we can write

gk(x) . mk = (x r1)(x r2) ... (x

where qk(x) is an integer polynomial.


Note that r1r2 . . Tk divides the constant term of gk(x) mk, which
equals 1- mk.
Since rnk > 1, 1 172k cannot be 0,

1 mk! rir2 - -

Now are distinct integers, and none of them isO. so

rlr2..rkl
hence
mk Ik/21! + 1.
Thi5 value of mk is attained by

gk(x) = 1)(x+1)(x-2)(x+2)
(x+ (_1)klk/2]) + Lk/2i! [k/2]! + 1.
Thus,
mk = Lk/2Th [k/2]! + 1.
72 4. Solutions to Advanced

Problem 59
Let x1 = 2 and
= xn +

1 1 1 1
1
22 Xi X2 2

Solution 59
Since x1 = 2 and
1 = 1),

is increasing.
Then 1 0.
Hence
1 1 _1 1

1 1) 1
or

+...+=1-
1

x1 x2
1 1 1

xn+11
Thus it suffices to prove that, for m N,

1 1 1
1
22
<1_ 22

or
1 (1)

We use induction to prove (1).


For ri = 1, X2 = x1 + 1 = 3 and (1) becomes 2 < 3 <4, which is
true.
Now suppose that (1) is true for some positive integer ii = k, i.e.,
22k1 22k
(2)

Then for ri = k + 1, the lower bound of (1) follows from


22k1 22k
Xk+2 1 = 1)
Solutions to Advanced Problems 73

Since xk4-1 is an integer, the lower bound of (2) implies that


22k
xk+1 and 1 1,

from which it follows that


22k (22k 22k+1
xk+2 1 = Xk+1(Xk+1 1) i)
desired.
This finishes the induction and we are done.

problem 60 [Iran 1997]


Suppose that f R+ is a decreasing function such that for all
:

z,y E R+,

f(x + y) + f(f(x) + 1(y)) = f(f(x + f(y)) + f(y + 1(x))).


Prove that f(f(x)) = x.
Solution 60
Setting y = x gives

f(2x) + f(2f(x)) = f(2f(x + f(x))).


Replacing x with f(x) yields

f(2f(x)) + f(2f(f(x))) = 1(21(1(x) + f(f(x)))).


Subtracting these two equations gives

1(21(1(x))) - f(2x) = f(2f(f(x) + f(f(x)))) - f(2f(x + 1(x))).


If 1(1(x)) > x, the left hand side of this equation is negative, so

f(f(x) +1(1(x))> f(x+ f(x))


and
f(x)+f(f(x)) <x 1-f(x),
acontradiction A similar contradiction occurs if f(f(x)) <x.
Thus f(f(x)) = x as desired.

Comment: In the original formulation I was meant to be a continous


function. The solution above shows that this condition is not necessary.
74 Solutions to Advanced

Problem 61 [Nordic Contest 1998]


Find all functions f : Q i Q such that
f(x+y)+f(x -y)2f(x)+2f(y)
for all x,y EQ.
Solution 61
The only such functions are 1(x) = kx2 for rational k. Any such function
works, since

f(x+y)+f(xy)=k(x+y)2+k(x--y)2
=kx2+2kxy+ky2+kx2 2kxy+ky2
= 2kx2 + 2ky2
= 21(x) + 21(y).
Now suppose f is any function satisfying
f(x+ y)+ f(xy) =2f(x)+2f(y).
Then letting x = y = 0 gives 21(0) = 41(0), so 1(0) = 0.
We will prove by induction that f(nz) = n2f(z) for any positive integer
ri and any rational number z.
The claim holds for ri = 0 and ri = 1; let ri 2 and suppose the claim
holds for n and n 2.
1

Then letting x = (n 1)z, y z in the given equation we obtain


f(riz) + f((n 2)z) = f((n 1)z + z) + f((n 1)z z)
=2f((n1)z)+2f(z)
so

f(nz) = 2f((ri 1)z) + 2f(z) f((ri 2)z)


= 2(n 1)21(z) + 2f(z) (n 2)21(z)
(2n24n+2+2n2+4n4)f(z)
= ri2f(z)

and the claim holds by induction.


Letting x = 0 in the given equation gives

f(y) + f(y) = 2f(0) + 2f(y)


so f(y) = 1(y) for all rational y; thus f(nz) = n2f(z) for all integers
n.
Solutions to Advanced Problems 75

Now let Ic = f(1): then for any rational number x = p/q,

q2f(x) = f(qx) = 1(p) = p2f(1) = kp2

5
1(x) = kp2/q2 = kx2.
Thus the functions f(x) = kx2. k E Q. are the only solutions.

Problem 62 [Korean Mathematics Competition 2000]


Let <a < 1.
Prove that the equation
x3(x+1)=(x-i-a)(2x--a)
has four distinct real solutions and find these solutions in explicit form.
Solution 62
Look at the given equation as a quadratic equation in a:
a2+ 3xa+2x2 x3 x4 =0.
The discriminant of this equation is
9x2 8x2 + 4x3 + 4x4 = (x + 2x2)2.
Thus
3x(x+2x2)
a
2

The first choice a = x + x2 yields the quadratic equation x2 x a = 0.


whose solutions are
(1'/1+4a)
2

The second choice a = 2x x2 yields the quadratic equation

x2 + 2x + a = 0,

whose solutions are

The inequalities

Show that the four solutions are distinct.


76 4. Solutions to Advanced

Indeed

reduces to
< 3 v'l + 4a
which is equivalent to

6V1 + 4a < 6 + 8a,


or 3a < 4a2, which is evident.

Problem 63 [Thurnament of Towns 1997]


Let a, b, and c be positive real numbers such that abc = 1.

Prove that
1 1 1
+ + <1.
a+b+1 b+c+1 c+a+1
Solution 63, Alternative 1
Setting x = a + b, y = b + c and z = c + a, the inequality becomes

x+1 +
y+l+ <1,
1 1 1

z+1
i.e.
1 1 x
y+l z+1 x+1'
i.e.
y+z+2 < x
(y+1)(z+1) x+1'
i.e.
xy+xz+2x+y+z+2 xyz+xy+xz+x,
i.e.
x+y+z+2 xyz,
i.e.
2(a+b+c)+2 (a+b)(b+c)(c-4-a),
i.e.
2(a + b + c) a2b + ab2 + b2c + be2 c2a + ca2.
By the AM-GM inequality,
(a2b-t-a2c+1) 3a.
Advanced Problems 77

Likewise,
(b2c+b2a+ 1) 3b

(c2a+c2b+ 1) 3c.
Therefore we only need to prove that
2(a+b+c)+3 3(a+b+c),
i.e.
3 a+b + c,
which is evident from AM-GM inequality and abc = 1.

Solution 63, Alternative 2


Let a = b= C = Then a1b1c1 = 1. Note that
(ai 0,
which implies that
a1b1(a1+bi).
Therefore,

1 1

a+b+1 =
1

aibi(ai + b1) +a1b1c1


a1b1c1
aibi(ai + b1 + c1)

Cl

a1

b1

a1+b1+c1
Adding the three inequalities yields the desired result.
78 4. Solutions to Advanced Problems

Problem 64 [AIME 1988]


Find all functions f, defined on the set of ordered pairs of positive inte..
gers, satisfying the following properties:
f(x,x)x, f(x,y)=f(y.x). (x+y)f(x.y)=yf(x,x-4-y).
Solution 64
We claim that f(x. y) = lcm(x. y). the least common multiple of x and
y. It is clear that
lcm(x, x) = x
and
lcm(x, y) = lcm(y, x).
Note that
y
lcm(x.y)=
gcd(x.y)
and
gcd(x.y) =gcd(x,x+y),
where gcd (u. v) denotes the greatest common divisor of u and v. Then

(x+y)lcm(x,y) =

x(x+y)
gcd(x,x+y)

= ylcm(x,x+y).
Now we prove that there is only one function satisfying the conditions of
the problem.
For the sake of contradiction, assume that there is another function
g(x, y) also satisfying the given conditions.
Let S be the set of all pairs of positive integers (x, y) such that f(x, y)
g(x, y), and let (in, ii) be such a pair with minimal sum m+ri. It is clear
that in n, otherwise
I (in, ii) = f(m. m) = in = g(in. in) = g(ni. n).

By symmetry (f(x, y) = f(y, x)), we can assume that n in > 0.


Note that
nf(rn, n in) = [in + (n m)]f(rn. ii in)
= (m ni)f (in, rn + (n in))
= (71. in)f(m., n)
Solutions to Advanced Problems 79

or nrn
f(rn.rirn)= f(rnn).
[,ikewiSe,
= ii
g(rn, n in) . g(rn. n).

Since f(rn. ii) g(m. n), f(rn. n in) g(rn. n in).


Thus (in. n rn) E S.
But (in. n in) has a smaller sum in + (n in) = n. a contradiction.
Therefore our assumption is wrong and f(x, y) lcm(x. y) is the only
solution.

problem 65 [Romania 1990]


Consider ii complex numbers zk. such that IzkI 1. k = 1.2
Prove that there exist c1.e2, . ..
. E (1. 1} such that, for any in ii,

I
+ e2z2 + + emzrnl 2.

Solution 65
Call a finite sequence of complex numbers each with absolute value not
exceeding 1 a green sequence.
Call a green sequence happy if it has a friend sequence
of is and is, satisfying the condition of the problem.
We will prove by induction on ii. that all green sequences are happy.
For n = 2. this claim is obviously true.
Suppose this claim is true when n equals some number in. For the case
of ii = rn + 1. think of the zk as points in the complex plane.
For each k. let ek be the line through the origin and the point corre-
sponding to zk. Among the lines some two are within 600 of
each other; suppose they are and with the leftover one being 4.
The fact that and are within 600 of each other implies that there
exists some number c9 {1, 1} such that z' = + e9z9 has absolute
value at most 1.
the sequence z'. z4. z5 is a k-term green sequence. so,
by the induction hypothesis, it must be happy: let c'. C4. C5 Ck+1
be its friend.
Let Ca 1.
Then the sequence is the friend of Induction is now
comp'ete
80 4. Solutions to Advanced Problems

Problem 66 [ARML 1997]


Find a triple of rational (a, b, c) such that

Solution 66
Let x = 1 and y = Then y3 = 2 and x = Note that

1 =y3 1 =(y 1)(y2+y+ 1).


and

2
3 3

3
x3 =yl= = (y+l)3
or

(1)
y+1
On the other hand,
3=y3+1=(y+1)(y2y+1)
from which it follows that
1 2 L1
(2)
y+l 3

Combining (1) and (2), we obtain

x=

Consequently,
(4 2 1

is a desired triple.
4
Solutions to Advanced Problems 81

problem 67 [Romania 1984)


Find the minimum of

(X2 + + +

where Xl.x2 are real numbers in the interval 1).

Solution 67
Since logs x is a decreasing function of x when 0 < a < and. since
(x 1/2)2 0 implies x2 x 1/4, we have

= xk+1 =

It follows that

+ (X3 + +

(logx2 logx3 logxi


2 + + + +
logx2

2n

by the AM-GM inequality.


Equalities hold if and only if

Problem 68 [AIME 1984]


Determine x2 + y2 z2 + w2 if

22_12 +2232+2252 +22_72 =1,

42_ 12 + 42_32 + 42_52 + 42_72 = 1,

z2 w2
62_12+62_32+ 62 52+62 72 1,

82_12 +8232+8252+8272=1.
82 4. Solutions to Advanced

Solution 68
The claim that the given system of equations is satisfied by
and w2 is equivalent to claiming that
z2 w2
t_12+t_32+i_52+t_72_l (1)

is satisfied by t = 4, 16. 36, and 64.


Multiplying to clear fractions, we find that for all values of t for which it
is defined (i.e., t 1, 9,25, and 49), (1) is equivalent to the polynomial
equation
P(t) = 0,

where

P(t) = (t 1)(t 9)(t 25)(t 49)


x2(t 9)(t 25)(t 49) y2(t 1)(t 25)(t 49)
z2(i l)(t 9)(t 49) w2(t 1)(t 9)(t 25).

Since deg P(t) = 4, P(t) = 0 has exactly four zeros t = 4, 16, 36, and 64,
i.e.,
P(t) = (t 4)(t 16)(t 36)(t 64).

Comparing the coefficients of t3 in the two expressions of P(t) yields

I +9+25+49+x2+y2+z2+w2 =4+16+36+64,
from which it follows that

+ y2 + z2 + w2 = 36.

Problem 69 [Balkan 1997]


Find all functions f : R IR such that

f(xf(x) + 1(y)) = (f(x))2 + y


for all x,y ER.
Solution 69
Let f(0) = a. Setting x = 0 in the given condition yields

I (f (y)) = a2 +
for all y E R.
Since the range of a2 +y consists of all real numbers, f must be surjective.
4
Solutions to Advanced Problems 83

Thus there exists b C R such that f(b) = 0.


getting x = b in the given condition yields

f(f(y)) = f(bf(b) + 1(y)) = (1(b))2 +y=


for all y C IL It follows that, for all x, y
(f(x))2 + y = f(xf(x) +
f [1 (1 (x))f(x) + f(y)] = I [f(x)f(f (x)) + yJ

= f(f(x))2 + y = x2 + y,
that is,
(f(x))2=x2. (1)
It is clear that f(x) = x is a function satisfying the given condition.
Suppose that f(x) x. Then there exists some nonzero real number c
such that f(c) = c. Setting x = cf(c) + f(y) in (1) yields

[f(cf(c) + 1(y))]2 = [cf(c) + 1(y)]2 = fc2 + 1(y)]2,


for all y C R, and, setting x = c in the given condition yields

f(cf(c)+f(y)) = (f(c))2+y =c2+y,


for all y C IL

that (f(y))2 = y2.


It follows that
fc2 +1(y)]2 = (c2 +y)2,
or

f(y)=y,
for all y C R, a function which satisfies the given condition.
Therefore the only functions to satisfy the given condition are 1(x) = x
or 1(x) = x, for x C R.

Problem 70
The numbers 1000, ,2999 have been written on a board.
Each time, one is allowed to erase two numbers, say, a and b, and replace
them by the number min(a, b)
After 1999 such operations, one obtains exactly one number c on the
board.
Prove that c < 1.
84 4. Solutions to Advanced

Solution 70
By symmetry, we may assume a b. Then
1. =
a

We have
11 1

from which it follows that the sum of the reciprocals of all the numbers
on the board is nondecreasing (i.e.. the sum is a monovariant).
At the beginning this sum is

s-++...+<-.
1000
1

1001 2999 c
1 1 1

where 1/c is the sum at the end. Note that, for I k 999,

1 1 4000 4000
+
2000k 2000+k 20002k2 > 20002 jij1jfJ

Rearranging terms in S yields

->
1

c 1000
11 1 i\
2999)
/1
2998)

> i,

or c < 1, as desired.

Problem 71 [Bulgaria 1998]


Let a1, a2 be real numbers, not all zero.
Prove that the equation

has at most one nonzero real root.


Solution 71
Notice that = + a2x is concave. Hence
45olutions to Advanced Problems 85

is concave.
Since f'(x) exists. there can be at most one point on the curve y = f(x)
with derivative 0.
Suppose there is more than one nonzero root.
Since x = 0 is also a root, we have three real roots x1 < x2 < x3. Ap-
plying the Mean-Value theorem to 1(x) on intervals [xl.X2] and [x2,x3],
we can find two distinct points on the curve with derivative 0, a contra-
diction.
Therefore, our assumption is wrong and there can be at most one nonzero
real root for the equation f(x) = n.

Problem 72 [Turkey 1998]


Let be the sequence of real numbers defined by a1 = t and

for n 1.
For how many distinct values of t do we have a1998 = 0?

Solution 72, Alternative 1


Let 1(x) = 4x(1 x). Observe that

= {0. 1}, f1(1) = {1/2}. 1]) = [0,1],


and [0,1).
Let = {x E R: = 0}; then

=
=
We claim that for alln 1, c [0. 1]. 1 E and

= + 1.

For n 1. we have

A1 = {x E R f(x) = 0} =
I
fO. 1},

and the claims hold.


NOW suppose n 1 and C [0. 1]. 1 e and = 2n_1 + 1. Then

8o
c [0. 1].
86 4. Solutions to Advanced

Since f(O) = f(1) = 0, we have = 0 for all n 1, so I e


Now we have

=
=
aE

= {x:f(x)=l}I+ > I{x:f(x)=a}I

= 1+ 2
aE
aE[O,1)
=
= 1+2(2n_1+1_1)
=
Thus the claim holds by induction.
Finally, a1998 = 0 if and only if I 1997(t) = 0, so there are 21996 + 1 such
values of t.
Solution 72, Alternative 2
As in the previous solution, observe that if f(x) E [0, 1] then x e [0, 1],
so if a1998 = 0 we must have t E [0. 1].
Now choose U e ir/2[ such that sin U =
Observe that for any e

f(sin2 (1 = 4sin2 = sin2

since = sin2 0. it follows that

a2 = sin2 20. a3 = sin2 40 a1ggg = sin2 219970.

Therefore

a1998 = 0 sin 219970 = 0 0


=
for some k e 7Z.

Thus the values of t which give a1998 = 0 are

sin2 (kir/21997),

k Z, giving 21996 + 1 such values of t.


to Advanced Problems

problem 73 [IMO 1997 short list]


(a) Do there exist functions 1 : R p R and g : R such that
f(g(x))=x2 and
for all x e
(b) Do there exist functions f R i R and g R R such that
f(g(x)) = x2 and g(f(x)) =
for all x E R?

Solution 73
(a) The conditions imply that f(x3) = f(g(f(x))) = [1(x)]2, whence
XE fi, 0, 1} ; x3 = x : f(x) = ; f(x) E {O, 1}.

Thus, there exist different a, b E fI, 0, 1} such that 1(a) = 1(b).


But then a3 = g(f(a)) = g(f(b)) = b3, a contradiction.
Therefore, the desired functions f and p do not exist.
(b) Let
I if lxi 1
p(x) = - In IxI if 0 < xi < I
1 0 ifx=0.
Note that g is even and al = bi whenever g(a) = g(b); thus, we
are allowed to define f as an even function such that
1(x) = where y is such that p(+y)
= x.
We claim that the functions f, p described above satisfy the condi-
tions of the problem.
It is clear from the definition of f that f(p(x)) = x2.
Now let y =
Then g(y) = x and

p(f(x)) = 9(y2)
(y2)Ifl(Y2) = y4Ifl Y = if i
= (yInY)4 if 0< y < 1
( 0 ify=0
= [9(y)]4
= x4.
88 4. Solutions to Advanced Problems

Problem 74 [Weichao Wu]


LetO< a1 a2 <b1

Suppose that there exists 1 k n such that a, for 1 i k and


b2 for 1> k.
Prove that
a1a2

Solution 74, Alternative 1


We define two new sequences. For i = 1.2 n, Let

= and
/
= .
Then
/ , ak
= ak = (a1
or
= (ak

Therefore

nak = a2 + + + + ... +
Applying the AM-GM inequality yields

(bib2.bnafl* =

from which the desired result follows.


Solution 74, Alternative 2
We define two new sequences. For = 1.2 n, Let

and

Then
(1)

Note that, for cy(x y)(y + c) 0


x x+c x>yandc>0:
>---.
yy+c,
45olutions to Advanced Problems

Setting x= y= and c = ak the above inequality implies that


aj/bi for i = 1. 2, , n. Thus,
.

Using (1) and the AM-GM inequality yields


F +... +
) = .
)

or
aia2
It is clear that the desired result follows from (2) and (3).

Problem 75
Given eight non-zero real numbers a1, a2 a8, prove that at least one
of the following six numbers: a1a3 + a2a4, aja5 + a2as, a1a7 + a2a8,
a3a5 + a4a5, a3a7 + a4a8, + a6a8 is non-negative.
Solution 75 [Moscow Olympiad 1978]
First, we introduce some basic knowledge of vector operations.
Let u = [a. b] and v = Im, nJ be two vectors.
Define their dot product u v = am + bn.
It is easy to check that
(i) vv = m2+n2 = 1v12, that is, the dot product of vector with itself
is the square of the magnitude of v and v v 0 with equality if
and only if v = [0.0];
(ii) u v = v u;
(iii) u (v + w) = u v + w. where w is a vector;
(iv) (cu) v = c(u . v), where c is a scalar.
.

When vectors u and v are placed tail-by-tail at the origin 0. let A and
.8 be the tips of u and v. respectively. Then = v u.
Let LAOB =0.
Applying the law of cosines to triangle AOB yields
= AB2
= 0A2-I-0B2-20A.OBcosO
= juj2 + v]2 21u!JvJ cosO.
90 4. Solutions to Advanced Problem

It follows that
(v u) (v
. u) = uu+v v 2IuIlvIcosO,

or
cosO=
IuHvI
Consequently, if 0 0 900, u v 0.
Consider vectors v1 = [ai, a2], v2 = [a3, a4], v3 = [a5, a6], and v4
[a7, as].
Note that the numbers a1a3+a2a4. a1a5+a2a6, a1a7+a2a8, a3a5+a4a6,
a3a7 + a4a8, a5a7 + a6a8 are all the dot products of distinct vectors Vj
and
Since there are four vectors, when placed tail-by-tail at the origin, at
least two of them form a non-obtuse angle, which in turn implies the
desired result.

Problem 76 [IMO 1996 short list]


Let a, b and c be positive real numbers such that abc = 1.

Prove that
ab bc ca
a5+b5+ab+b5+c5+bc+c5+a5+ca 1.
Solution 76
We have
a5 + b5 a2b2(a + b),
because
(a3 b3)(a2 b2) 0,
with equality if and only if a = b. Hence
ab ab
a2b2(a+b)+ab
1

ab(a+b)+1
abc
ab(a+b+c)
C

a+b+c
Likewise,
bc a
b5 + c5 + bc a+b+c
45olutions to Advanced Problems 91

and
b

+ a5 + ca a + b + c
c5

the last three inequalities leads to the desired result.


Equality holds if and only if a = b = c = 1.

Comment: Please compare the solution to this problem with the


second solution of problem 13 in this chapter.

Problem 77 [Czech-Slovak match 1997]


Find all functions f R * R such that the equality

f(f(x)+y) =f(x2 y) +4f(x)y


holds for all pairs of real numbers (x, y).

Solution 77
Clearly. f(x) = x2 satisfies the functional equation.
Now assume that there is a nonzero value a such that f(a) a2.
x2 1(x)
Let y = in the functional equation to find that

(f(x)+x2) (f(x)+x2)
=

2f(x)(x2 1(x) or 1(x) = x2.


In both cases. f(0) = 0.
Setting x = a, it follows from above that either f(a) = 0 or 1(a) = 0 or
f(a)=a2.
The latter is false, so f(a) = 0.
Now, let x = 0 and then x = a in the functional equation to find that

1(u) = f(y), 1(y) = f(a2

and so

1(n) = f(y) = f(a2 + y);


that is, the function is periodic with nonzero period a2.
Let y= a2 in the original functional equation to obtain
f(f(x)) = 1(1(x) + a2) = f(x2 a2) + 4a2f(x) = f(x2) + 4a2f(x).
However, putting y = 0 in the functional equation gives 1(1(x)) = 1(x2)
for all x.
92 4. Solutions to Advanced Problems

Thus, 4a2f(x) = 0 for all x. Since a is nonzero, 1(x) = 0 for all x.


Therefore, either 1(x) = x2 or f(x) = 0

Problem 78 [Kvantj
Solve the system of equations:
3x y
x+ =3
x2 + y2
x+3y =0.
x2 + y2

Solution 78, Alternative 1


Multiplying the second equation by i and adding it to the first equation
yields
(3xy)(x+3y)i
=3,
or
i(xyi)
x2+y2 x2+y2
Let z = x + yi. Then
!_ xyi
z x2 + y2
Thus the last equation becomes
3i
z+ =
z
or
z23z+(3i) =0.
Hence
3v'34-4i 3(1+2i)
2 2

that is, (x,y) = (2,1) or (x,y) = (1,i).


Solution 78, Alternative 2
Multiplying the first equation by y, the second by x, and adding up yields
(3x y)y (x + 3y)x
2xy + =
x2 + y2

or 2xy 1 = 3y. It follows that y 0 and


3y + I
45olutions to Advanced Problems

this into the second equation of the given system gives

[(3y+ 1)2 (3Y+ 1)


+ = o.

or
4y4 3y2 1 = 0.

it follows that y2 1 and that the solutions to the system are (2, 1) and
(1,i).

problem 79 [China 1995]


Mr. Fat and Mr. Taf play a game with a polynomial of degree at least 4:

+ ... +_x + i.
They fill in real numbers to empty spaces in turn.
If the resulting polynomial has no real root. Mr. Fat wins; otherwise, Mr.
Taf wins.
If Mr. Fat goes first, who has a winning strategy?
Solution 79
Mr. Taf has a winning strategy.
We say a blank space is odd (even) if it is the coefficient of an odd (even)
power of x.
First Mr. Taf will fill in arbitrary real numbers into one of the remaining
even spaces, if there are any.
Since there are only n 1 even spaces, there will be at least one odd
Space left after 2n 3 plays, that is, the given polynomial becomes

p(x) = q(x) +_x2t_1,

where s and 2t I are distinct positive integers and q(x) is a fixed


Polynomial.
We claim that there is a real number a such that

p(x) = q(x) + +_x2t1


Will always have a real root regardless of the coefficient of
Then Mr. Taf can simply fill in a in front of x8 and win the game.
4. Solutions to Advanced

Now we prove our claim. Let b be the coefficient of x2t_1 in p(x). Note
that

+ p(l)
= + 2s_2t+la + b) + [q(1) + (1)8a

= + q(_1)) + +

Since s 2t 1 2s2t+1 + (_1)8 o.


Thus
1
+ q(1)
a=
2s2t+1 + (_1)8
is well defined such that a is independent of b and

+p(l) = 0.

It follows that either p(l) = p(2) = 0 or p(l.) and p(2) have different
signs, which implies that there is a real root of p(x) in between 1 and
2.
In either case, p(x) has a real root regardless of the coefficient of
as claimed.
Our proof is thus complete.

Problem 80 [IMO 1997 short list}


Find all positive integers k for which the following statement is true: if
F(x) is a polynomial with integer coefficients satisfying the condition
for c=0,1
then F(O) = F(1) = ... = F(k + 1).
Solution 80
The statement is true if and only if /c 4.
We start by proving that it does hold for each Ic 4.
Consider any polynomial F(x) with integer coefficients satisfying the
inequality 0 F(c) Ic for each c e {0. 1 k + 1}.
Note first that F(k + 1) = F(0), since F(k + 1) F(0) is a multiple of
Ic + 1 not exceeding Ic in absolute value.
Hence
F(x) F(0) = x(x Ic 1)G(x).
45olutions to Advanced Problems

where G(x) is a polynomial with integer coefficients.


consequently,

k jF(c) F(O)j = c(k + I c)IC(c)I

for each c E 2 k}.


The equality c(k + 1 c) > k holds for each cC {2, 3 k 1}, as it is
equivalent to (c 1)(k c) > 0.
Note that the set f2. 3 k 1} is not empty if k 3, and for any c in
this set. (1) implies that < 1.
Since C(c) is an integer, C(c) 0.
Thus

F(x)F(0)=x(x2)(x3)''(xk+1)(xk1)H(x). (2)

where H(x) is a polynomial with integer coefficients.


To complete the proof of our claim, it remains to show that 11(1) =
H(k) = 0.
Note that for c = 1 and c = k, (2) implies that
Ic IF(c) F(0)I = (Ic 2)! . Ic IH(c)I.

Fork4,(k2)!>1.
Hence H(c) = 0.
We established that the statement in the question holds for any k 4.
But the proof also provides information for the smaller values of Ic as
well.
More exactly, if F(x) satisfies the given condition then 0 and k + 1 are
roots of F(x) and F(0) for any Ic and if Ic 3 then 2 must also be
a root of F(x) F(0).
Taking this into account, it is not hard to find the following counterex-
amples:
F(x)=x(2x) fork=1.
F(x) = x(3 x) for Ic = 2.
F(x)=x(4x)(x2)2 for k=3.
96 4. Solutions to Advanced Problems

Problem 81 [Korean Mathematics Competition 2001J


The Fibonacci sequence is given by

(n.EN).
Prove that
i'3 r'3
L 2n+2 2n2

9
for all n 2.
Solution 81
Note that

= = =

whence
=0 (1)

for all n 2.
Setting a = b= and c = in the algebraic identity

abbcca)

gives
rv,n3
htf2n r,3 n3

n

Applying (1) twice gives

=
2 2

The desired result follows from

tlr2n+2r2nr2n_2 r'3 r' i r' r' r'2


r2n)\


4. Solutions to Advanced Problems 97

problem 82 [Romania 1998]


Find all functions u : R * R for which there exists a strictly monotonic
function f : R * R such that

f(x + y) = f(x)u(y) + f(y)


for all x,y ER.
Solution 82
The solutions are u(x) = ax. a E R.
To see that these work, take f(x) = x for a = 1.

1, take 1(x) =a 1; then


f(x + y) = 1 = (ax 1 = + f(y)
for all x, y E R.
Now suppose u R * R, f R R are functions for which f is strictly
monotonic and f(x + y) = f(x)u(y) + f(y) for all x, y E R.
We must show that u is of the form u(x) = ax for some a E R+. First,
letting y = 0, we obtain f(x) = f(x)u(0) + f(0) for all x E R.
Thus, n(O) 1 would imply f(x) = f(O)/(1 u(O)) for all x, which
would contradict the fact that f is strictly monotonic, so we must have
u(0) = 1 and f(0) = 0.
Then f(x) 0 for all x 0.
Next, we have

f(x)u(y) + f(y) = f(x + y) = f(x) + f(y)u(x),


or
f(x)(u(y) 1) = f(y)(u(x) 1)

for all x,y ER. That is,


u(x) I u(y) 1

f(x) 1(y)
for all xy 0.
It follows that there exists C E R such that
u(x) I
=
f(x)
for all x 0.
Thus, u(x) = 1+Cf(x) for x 0: sinceu(0) = 1, f(0) = 0, this equation
also holds for x = 0.
98 4. Solutions to Advanced Problem

If C = 0, then u(x) = 1 for all x, and we are done.


Otherwise, observe

u(x+y) = 1+Cf(x+y)
= + Cf(x)u(y) + Cf(y)
1

= u(y)+Cf(th)u(y)
= u(x)u(y)

for all x,y cR.


Thus u(nx) = u(x) for all n Z, x R.
Since u(x) = 1 + Cf(x) for all x, u is strictly monotonic, and u(x) =
1/u(x) for all x, so u(x) > 0 for all x as u(O) = 1.
Let a = u(1) > 0; then u(n) = a for all n N, and
u(p/q) = (u(p))lk =
for allpe Z, q E N, so u(x) = ax for allxE Q.
Since u monotonic and the rationals are dense in
is we have u(x) = ax
for all x E R.
Thus all solutions are of the form u(x) = ax, a E

Problem 83 [China 1986]


Let z1 Z2,
, be complex numbers such that
,

Prove that there exists a subset S of {zi, Z2,... , such that

zS

Solution 83, Alternative 1


Let 2, and 3 be three rays from origin that form angles of 60, 180,
and 300, respectively, with the positive
For i = 1,2,3, let 7Z, denote the region between and (here 4 =
including the ray Then

1= jZkj+ > ZkH !ZkI


ZkEl?..3

By the Pigeonhole Principle, at least one of the above sums is not less
than 1/3.
4. Solutions to Advanced Problems

Sayit's (otherwise, we apply a rotation, which does not effect the


magnitude of a complex number), Let zk = xk + 1Yk. Then for Zk e 7Z3,
Xk XkI ZkI/2.

>

as desired.
Solution 83; Alternative 2
We prove a stronger statement: there is subset S of {zi, z2, .. . , z,.,} such
that

zES

For I k n, let Zk Xk + iyk. Then


1 =

=
>
XkO Xk<O yk<O

By the Pigeonhole Principle, at least one of the above sums is not less
than 1/4. By symmetry, we may assume that

Xk>O XkO

Consequently,

XkO

Comment: Using advanced mathematics, the lower bound can be


further improved to 1/ir.

Problem 84 [Czech-Slovak Match 1998]


A polynomial P(x) of degree n 5 with integer coefficients and n distinct
integer roots is given.
Find all integer roots of P(P(x)) given that 0 is a root of P(x).
100 4. Solutions to Advanced Problems

Solution 84
The roots of P(x) are clearly integer roots of P(P(x)); we claim there
are no other integer roots.
We prove our claim by contradiction. Suppose, on the contrary, that
P(P(k)) = 0 for some integer k such that P(k) 0.
Let
P(x) =a(xri)(x r2)(xr3).. .(x rn),
where a.rl,r2,...,rfl are integers,

Since P(k) 0,we must have 1k 1 for all i.


Since the are all distinct, at most two of 1k r2j. k r31, 1k equal
1, so

Ia(kr2).(krn_i)I
and IP(k)l

Also note that P(k) = for some i0, so P(k)!


Now we consider the following two cases:

1. 1k! rn!. Then P(k)I rn)! > rn!, a contradic-


tion.
2. 1k! < rn!, that is, 1 1k! 1. Let a. b, c be real numbers,
a b. For xe [a, b}, the function

f(x) =x(cx)
reaches its minimum value at an endpoint x = a or x = b, or at
both endpoints.
Thus

k(k rn)I = k!!rn kI !k!(!Tn! 1k!) 1.

It follows that

rn! P(k)! 1),

which implies that Tn! K 2. Since n 5, this is only possible if


P(x) = (x + 2)(x + 1)x(x 1)(x 2).

But then it is impossible to have k and 1k! rn!, a contra-


diction.
4
Solutions to Advanced Problems 101

Thus our assumption was incorrect, and the integer roots of P(P(x)) are
exactlY all the integer roots of P(x).

problem 85 [Belarus 1999}


Two real sequences x1, x2 and Y2 are defined in the following
way.

and
yn
1 + +
for all n 1. Prove that 2 < XnYn <3 for all n> 1.
Solution 85, Alternative 1
Let = 1 and note that the recursion for is equivalent to

Zn+1 = Zn + +
Also note that Z2 = = x1; since the and satisfy the same
recursion, this means that Zn = Xn_1 for all n> 1.
Thus,
XnYn =
Xn
=
2n
.
Xn
Xn_1
Note that
>

Thus 2Xn_i and XnYn > 2, which is the lower bound of the desired
inequality.
Since Xn5 are increasing for n> 1, we have

= 3>
which implies that
2Xn_i >

Thus 3Xn_i > Xn. which leads to the upper bound of the desired inequal-
ity.

Solution 85, Alternative 2


Setting = cot 0,, for 0 < 0,, <900 yields

= cot + + cot2 = cot + = cot


102 4. Solutions to Advanced Problen

300
Since = 30, we have in general Similar calculation shows
that
tan 2

1 tan

tan 0. tan2 is positive and XnYn > 2.


And since for n> 1 we have < 30. we also hare

tan2 <

so that XnYn <3.


Comment: From the closed forms for and in the second solution,
we can see the relationship

1
yn =
xfll

used in the first solution.

Problem 86 [China 1995]


For a polynomial P(x), define the difference of P(x) on the interval 4 b)
({a,b), (a,b), (a,bJ) as P(b) P(a).
Prove that it is possible to dissect the interval 9. 1] into a finite number
of intervals and color them red and blue alternately such that, for every
quadratic polynomial P(x), the total difference of P(x) on red intervals
equal to that of P(x) on blue intervals.
is

What about cubic polynomials?


Solution 86
For an interval i, let denote the difference of polynomial P on i.

For a positive real number c and a set S R, let S + c denote the set
obtained by shifting S in the positive direction by c.
We prove a more general result.

Lemma
Let be a positive real number, and let k be a positive integer. It is
always possible to dissect interval Ik [0. into a finite number of
such that, for every
intervals and color them red and blue alternatively
polynomial P(x) with degP k, the total difference of P(x) on the red
intervals is equal to that on the blue intervals.
45olutions to Advanced Problems 103

proof
We induct on k.
For k = 1, we can just use intervals and (t.2t]. It is easy to see
that a linear or constant polynomial has the same difference on the two
intervals.
Suppose that the statement is true for /c = n, where n is a positive
integer; that is. there exists a set of red disjoint intervals and a set
of blue disjoint intervals such that fl = @. U = and,
for any polynomials P(x) with deg P n. the total differences of P on
is equal to that of P on
consider polynomial f(x) with deg f n + 1. Define
g(x) = f(x + and h(x) = f(x) g(x).
Then deg h n. By the induction hypothesis,

or
Arf+
It follows that
Arf,

where

11-n+1


D L)ii(D T
)flD\

and = U +
(If and both contain the number that number may be
removed from one of them.)
It is clear that and form a dissection of 'n+l and, for any
Polynomial f with degf n + 1, the total difference of f on is
equal to that of f on
The only possible trouble left is that the colors in might not
be alternating (which can happen at the end of the and the beginning

But note that if intervals i1 = [a1, b1 J and i2 = [b1, c1 } are in the same
Color, then
+ =
Wherej3 = [ai,ci].
104 4. Solutions to Advanced Problems

Thus. in we can simply put consecutive same color intervals


into one bigger interval of the same color.
Thus, there exists a dissection
=
such that, for every polynomial f(x) with deg f n + 1.

This completes the induction and the proof of the lemma. 0


Setting first = and then = in the lemma, it is clear that the
answer to each of the given questions is

Problem 87 [USSR 1990]


Given a cubic equation
S3 + _X2 + _X + = 0,
Mr. Fat and Mr. Taf are playing the following game.
In one move, Mr. Fat chooses a real number ana Mr. Taf puts it in one
of the empty spaces.
After three moves the game is over.
Mr. Fat wins the game if the final equation has three distinct integer
roots.
Who has a winning strategy?
Solution 87
Mr. Fat has a winning strategy.
Let the polynomial be x3 + ax2 + bx + c. Mr. Fat can pick 0 first. We
consider the following cases:
(a) Mr. Taf chooses a = 0, yielding the polynomial equation
x3 + bx + c = 0.
Mr. Fat then picks the number (inrip)2, where in. n, and p are
three positive integers such that
m2 + n2 =

If Mr. Taf chooses b = (n2rip)2, then Mr. Fat will choose c = 0.


The given polynomial becomes
x(x n2np)(x + rnnp).
4golutions to Advanced Problems

If Mr. Taf chooses c = (rnnp)2. then Mr. Fat will choose

b = ?TL2fl2 fl2p2 p27772.

The given polynomial becomes

(x + rn2)(x + n2)(x p2).

(b) Mr. Taf chooses b = 0. yielding the equation

x3 + ax2 + c = 0.

Mr. Fat then picks the number

m2(m + 1)2(rn2 -+ m +

where rn is an integer greater than 1.


If Mr. Taf chooses

a = rn2(m + 1)2(m2 + m +
then Mr. Fat can choose

c = m8(in + 1)8(in2 + rn + 1)6.


The polynomial becomes

(x rnp)[x (rn 1)pJ[x + m(rn + 1)pJ,

where
p = m2(n2 + 1)2(rn2 + rn + 1)2.

If Mr. Taf chooses

c=m2(m+ 1)2(m2 +m+ j)3,


then Mr. Fat can choose

a= (m2 +m+ 1)2.

The polynomial becomes

(x+mq)[x(rn+1)qffxm(m+
where
q = in2 + in + 1.
106 4. Solutions to Advanced Problems

(c) Mr. Taf chooses c = 0.

Then the problem reduces to problem 40 of the previous chapter.


Mr. Fat needs only to pick two integers a and b such that

ab(a 1)(b1)

and a+b= 1.
The polynomial becomes either x(x 1)(x a) or x(x 1)(x b).

Our proof is complete.


Below is an example of what Mr. Fat and Mr. Taf could do:
F T F T F Roots
0 a 3600 b 0 60,0.60
c 481 16, 9,25
b
4.9.73 a .38.76 8.27.49,
4. 27 . 49,
89. 49
c 49 14, 21,42
c 2 a 3 3,0,1
" " "
b 3 0,1.2

Problem 88 [Romania 1996]


Let n> 2 be an integer and let f : R2 R be a function such that for
any regular n-gon A1A2. . .

f is the zero function.


Solution 88
We identify R2 with the complex plane and let c =
Then the condition is that for any z C C and any positive real t,

In particular, for each of k 1,. .. , n, we obtain


4. Solutions to Advanced Problems 107

gumming over we have

,n=1 k=1

Form = n the inner sum is nf(z); for other m, the inner sum again runs
over a regular polygon, hence is 0.
Thus f(z) = 0 for all z E C.
Problem 89 [IMO 1997 short list]
Let p be a prime number and let f(s) be a polynomial of degree d with
integer coefficients such that:
(i) 1(0) = 0, f(1) = 1;

(ii) for every positive integer n, the remainder upon division of 1(n)
by p is either 0 or 1.
Prove that dp 1.
Solution 89, Alternative 1
For the sake of the contradiction, assume that d p 2.
Then by Lagrange's interpolation formula the polynomial f(s) is
determined by its values at 0, 1, . . ., p 2; that is,


1 (x)
1 (1) .. (k p +2)
k=O

p-2
k)5 (x xk (x P+2)
k!(_1)P-k (p k 2)!
k=O

Setting x = p 1 gives

k)
f(p 1) = >f(k)

=
(_1)Pkk!
p2
(modp).

It follows that
S(f) := f(o) + f(1) + ... + f(p -1) 0 (mod p) (1)
108 4. Solutions to Advanced Problems

On the other hand, (ii) implies that S(f) j (mod p), where j
the number of those Ic E 1 p 1} for which 1(k) 1 (mod p)
But (i) implies that 1 j < p 1.
So S(f) 0 (mod p), which contradicts (1).
Thus our original assumption was wrong. and our proof is complete.
Solution 89, Alternative 2
Again, we approach the problem indirectly.
Assume that d p 2, and let

1(x) = + + a1x + a0.

Then
pi pi p2 p2 pi p2
S(f) = 1(k) = = =
k=0 k=O i=0 i=0 k=O i=0

pi
where =

(modp)foralli=0,1 p2.
We use strong induction on I to prove our claim.
The statement is true for i 0 as So = p.
Now suppose that So S1 .. 0 (mod p) for some 1 i
p2. Note that
p pi pI

k=Oj=0 j0

(I + (mod p)

Since 0 < i + 1 <p, it follows that 0 (mod p). This completes the
induction and the proof of the claim. Therefore,

S(f) 0 (mod p).


=
The rest is the same as in the first solution.
4Solutions to Advanced Problems 109

problem 90
Let n be a given positive integer.
Consider the sequence a0, a1. with a0 = and

ak = ak_I +
n
1

for k = 1, 2, , n.
Prove that
1
1 < 1.
Ti

Solution 90, Alternative 1


We prove a stronger statement: For k = 1. 2

71
(1)

We use induction to prove both inequalities.


We first prove the upper bound, For k = 1, it is easy to check that
1
a1 = + =
1 2n+1 n
2 4n 4n 2ni
Suppose that
<
2n k'
for some positive integer k <n. Then
ak
= (n-f-ak)

< 271_k(Ti+271_k)
n(2nk+1)
(2nk)2
<
2nk1'

(2n k+ 1)(2n ki) = (2nk)2 1< (2n k)2


Thus our induction step is complete. In particular, for k = n 1,

n
afl=ak+1< =1,
2n (n 1) I
110 4. Solutions to Advanced Problems

as desired.
Now we prove the upper bound. For k = 1, it is easy to check that
2n+1 n+1
=
4n
Suppose that
n+1
ak>
2ri k + 2'
for some positive integer k < n. Then
ri + 1 (n -I- 1)2
ak+l ak + > 2n_k+2+n(2n_k+2)2
n

It follows that
n+1 n+1 (ri.l-1)2
ak+i_2k+l (2n_k+l)(2n_k+2)+n(2n_k+2)2
n+1 "ri+l 2nk+2\
=

n+1 /1 1

2nk+2)2 2n_k+1) >0.


This complete the induction step. In particular, for n = k 1, we obtain

n+1 72+1
2n(n1)+1 n+2 n+2 ri

as desired.

Solution 90, Alternative 2


Rewriting the given condition as
1 Ti 1 1

ak a

ak rt+ak_1

It is clear that aks are increasing.


Thus
I

2
4. Solutions to Advanced Problems 111

Thus (2) implies that


1 1
<
ak_i
1

n ak
for k = 1, 2, . .. , fl
Telescoping summation gives
1 1
<1
a0

or
1 1
> 1 = 21 = 1,
a0
that is, <1, which gives the desired upper bound.
Since < 1, and, since aks are increasing, = < < 1 for
k=1,2,...,n.
Then (2) implies
1 1 1
>
ak n+ak_1 n+1
fork=1,2,...,ri.
Telescoping sum gives
1 1
-->
a0 n+1
or
1<1 72 n+2
ao n+ln+1
that is,
n+1 =1 1 >1,
n+2 n-f-2 n
which is the desired lower bound.

Problem 91 [IMO 1996 short list]


Let a1, a2,.. . , be nonnegative real numbers. not all zero.
(a) Prove that .. = 0 has precisely one
positive real root R.
(b) Let A = and B = jaj.
Prove that AA RB.
112 4. Solutions to Advanced Problem

Solution 91
(a) Consider the function
a1 a2

Note that f decreases from oo to 0 as x increases from 0 to oc.


Hence there is'a unique real number R such that f(R) = 1, that is,
there exists a unique positive real root R of the given polynomial.
(b) Let c3 = ad/A.
Then c3s are non-negative and c2 = 1.

Since ln x is a convex function on the interval (0, oo), by Jensen's


inequality,

Ecj ln =-In(f(R))=0.

ft follows that
c3(lnA+jlnR) 0
j=1
or

c3 = a3 /A, we obtain the desired inequality.

Comment: Please compare the solution of (a) with that of the problem
15 in the last chapter.

Problem 92
Prove that there exists a polynomial P(x, y) with real coefficients such
that P(x, y) 0 for all real numbers x and y, which cannot be written
as the sum of squares of polynomials with real coefficients.
Solution 92
We claim that
P(x,y) = (x2 +y2 1)x2y2 +
is a polynomial satisfying the given conditions.
First we prove that P(x, y) 0 for all real numbers x and y.
4. Solutions to Advanced Problems 113

If x2 + y2 I 0, then it is clear that P(x,y) > 0; if x2 + y2 1 <0,


then applying the AM-GM inequality gives

or
(x2 + y2 l)x2y2

It follows that P(x,y) 0.


We are left to prove that P(x, y) cannot be written as the sum of squares
of polynomials with real coefficients.
For the sake of contradiction, assume that

P(x,y) =

Since degP = 6, 3.
Thus

=
+ + C2y2 + + I2y +

Comparing the coefficients, in P(x, y) and y)2, of terms x6


and y6 gives
=0,

orA' =D2=Oforalli.
Then, comparing those of x4 and y4 gives

= =

Next, comparing those of x2 and y2 gives

= 0,

or H, = for all
Thus,
= + F,.xy +
114 4. Solutions to Advanced Problems

But, finally, comparing the coefficients of the term x2y2, we have

= -1,

which is impossible for real numbers


Thus our assumption is wrong, and our proof is complete.

Problem 93 [IMO 1996 short list]


For each positive integer n, show that there exists a positive integer k
such that
k = f(x)(x + 1)2n +
g with integer coefficients, and find the smallest
such /c as a function of n.
Solution 93
First we show that such a k exists.
Note that x + 1 divides 1 Then for some polynomial a(s) with
integer coefficients, we have

(1 + x)a(x) = 1 = 2 (1 +

or
2=(1+x)a(x)+(1 +x272).
Raising both sides to the (2n)th power, we obtain

= (1 + + (1 +

where b(s) is a polynomial with integer coefficients.


This shows that a k satisfying the condition of the problem exists. Let
/c0 be the minimum such k.
Let 2n = 2r . q, where r is a positive integer and q is an odd integer.
We claim that =
First we prove that 2q divides k0. Let t = Note that + 1
(5t + 1)Q(x), where

Q(x) = 5t(q_l)
+ ... + 1.

The roots of + 1 are


((2m1)ir\ . f(2m1)ir\
.
m=1,2,...,t;
),
Solutions to Advanced Problems 115

that is,
R(x)=xt+l=(x_wi)(x_w2)..(x_wt).
Let f(s) and g(x) be polynomials with integer coefficients such that
k0 f(x)(x+1)272+g(x)(x272+1)
= f(x)(x + 1)2n + g(x)Q(x)(xt + 1).
It follows that

f(Wm)(Wm+1)Th=ko, (1)

Since r is positive, t is even. So

2=R(1) = +w2)(1+wt).
Since is a symmetric polynomial in .. ,

with integer coefficients, it can be expressed as a polynomial with integer


coefficients in the elementary symmetric functions in w1, w2,. .. ,

and therefore
F = f(wl)f(w2) .

is an integer.
Taking the product over m = 1,2,... , t, (1) gives 2272F = ku or =
It follows that 2q divides k0.
It now suffices to prove that k0
Note that Q(1) = 1.
It follows that
Q(x) =(x+1)c(x)+1,
where c(s) is a polynomial with integer coefficients.
Hence
(x + 1)2n = Q(x)d(x) + 1, (2)

for some polynomial d(x) with integer coefficients.


Also observe that, for any fixed m,

Thus
(1 +(iJm)(1 (1 +w2t_1) = R(1) = 2,
and writing
116 4. Solutions to Advanced Problems

we find that for some polynomial h(x), independent of m. with integer


coefficients such that
(1 + w)th(w) = 2.
But then (x + 1)h(x) 2 is divisible by xt + 1 and hence we can write

(x + 1)h(x) = 2+ (xt + 1)u(x),


for some polynomial u(x) with integer coefficients.
Raising both sides to the power q we obtain

(x + = 2q + (xt + 1)v(x), (3)

where v(x) is a polynomial with integer coefficients.


Using (2) and (3) we obtain

(x + + 1)v(x)
= Q(x)d(x)(xt + 1)v(x) + (xt + 1)v(x)
2q,
= Q(x)d(x)(xt -I- 1)v(x) + (x +
that is,
2q
= fi(x)(x + + g1(x) + 1),
where (x) and 91(x) are polynomials with integer coefficients.

Hence k0 < 2q, as desired.


Our proof is thus complete.

Problem 94 [USAMO 1998 proposal, Kiran Kedlaya]


Let x be a positive real number.
(a) Prove that
(ni)! _!
(b) Prove that

(ni)! 1
4. Solutions to Advanced Problems 117

Solution 94
We use infinite telescoping sums to solve the problem.
(a) Equivalently, we have to show that

that

n!x

(ni)! n!

and this telescoping summation yields the desired result.


(b) Let
(n-i)!
f( )-
X

Then, by (a), 1(x)

In particular, 1(x) converges to 0 as x approaches oo, so we can


write f as an infinite telescoping series

= +k -1) - f(x + k)]. (1)

On the other hand, the result in (a) gives

f(x-1)-f(x) =

(ni)!
x1(x+1)(x+n)
118 4. Solutions to Advanced Problems

Substituting the la.st equation to (1) gives

as desired.

Problem 95 [Romania 1996]


Let n 3 be an integer, and let
XcS={1,2,,,.,n3}
be a set of 3n2 elements.
Prove that one can find nine distinct numbers (i = 1,2,3) in X
such that the system
aix+biy+ciz = 0

a2x+b2y+c2z = 0
a3x+b3y+c3z = 0

has a solution (xo, Yo, zo) in nonzero integers.


Solution 95
Label the elements of X in increasing order Xi << and put

X1 = {x1,...
X2 =
X3 =
Define the function f : X1 x X2 x X3 S x S as follows:

f(a, b, c) = (b a, c b).

The domain of f contains n6 elements,


The range of f, on the other hand, is contained in the subset of S x S
of pairs whose sum is at most n3, a set of cardinality

6
1

By the Pigeonhole Principle, some three triples (a2, cj) (i = 1,2,3)


map to the same pair, in which case x = = ci a1,z = a1 bi
is a solution in nonzero integers.
4. Solutions to Advanced Problems 119

Note that cannot equal since X1 and X2 are disjoint, and that
ai a2 implies that the triples (ai,bi,c1) and (a2,b2,c2) are identical,
a contradiction.
Hence the nine numbers chosen are indeed distinct.

problem 96 [Xuanguo Huang]


Let n 3 be an integer and let x1, x2, , be positive real numbers.
Suppose that
= 1.

Prove that
(1 1

Solution 96
By symmetry, we may assume that x1 x2 We have the
following lemma.
Lemma

1 + 1 + x3

Prooft Since n 3, and, since

= 1,
1

we have
1 1

or
>
It follows that x2x3> 1. Thus


1+x2 1+x3 (1+x1)(1+x3)

-
(1+x2)(1+x3)

0,
120 4. Solutions to Advanced Problems

as desired. o
By the lemma, we have

>
1+xi 1+x2

and, since

it follows by the Chebyshev Inequality


1
1 (1

By the Cauchy-Schwartz Inequality, we have

or
(2)

Multiplying by

on both sides of (2) and applying (1) gives

which in turn implies the desired inequality.


4. Solutions to Advanced Problems 121

problem 97
Let X1,X2,. . .be distinct real numbers.
Define the polynomials

P(x) =
and
Q(x)=P(x)( /1
\XXj
+
1

XX2
+..,+
Let Y1,Y2 Yn1 be the roots of Q.
Show that
minlx,,x31 <minly2y71.

Solution 97
By symmetry, we may assume that

d= minly, Yjl=

Let 8k = Xk, for k = 1.2,..., n.


By symmetry, we may also assume that .s1 < < i.e., x1 >

For the sake of contradiction, assume that

d minlx2 = minx, = mins3 (1)


i(j
Since P has no double roots, it shares none with Q.
Then
1 1 1
(\Y,X1 +
yjX2
+ = Q(Yi) 0,

or
1 1 1
+ =0.
YiX1 Y%X2
In particular, setting i = 1 and i = 2 gives

(2)

We claim that there is a k such that sk(sk + d) <0, otherwise, we have


1 1

Sk+d 8k
4. Solutions to Advanced Problems
122

for. all k, which in turn implies that

which contradicts (2).


< 0 < Sk+d.
Let j be the number of ks such that sk(sk+d) < 0, that is, Sk
A simple but critical fact is that 8k + d and have the same sign. In
fact, suppose that

then

Then > 0 if and only if /c > i + 1, that is Sk + d> 0.


From (1), we obtain Sk + d Sk+3, and, since sk + d and 3k+3 have the
same sign, we obtain
1 >1
Sk + d 8k+j
for all k = . ,n j. Therefore,

or
(3)

Also note that


E!<o<E 1
k=1
+
Sk d

Adding (3) and (4) yields


Ti fl

which contradicts (2).


Thus our assumption is wrong and our proof is
4. Solutions to Advanced Problems 123

Problem 98 [Romania 1998]


Show that for any positive integer n, the polynomial

f(x)=(x2+x)2"+l
cannot be written as the product of two non-constant polynomials with
integer coefficients.
Solution 98
Note that 1(x) = g(h(x)), where h(x) = x2 + x and 9(y) = y2" + 1.
Since

and is even for 1 k 1, g is irreducible, by Eisenstein's


criterion.
Now let p be a non-constant factor of f, and let r be a root of p.
Then g(h(r)) = f(r) = 0, so .s h(r) is a root of g.
Since s = r2 -f- r C Q(r), we have Q(s) C Q(r), so

deg p deg(Q(r)/Q) deg(Q(s)/Q) = deg g=


Thus every factor of f has degree at least T'.
Therefore, if f is reducible, we can write f(x) = A(x)B(x) where A and
B have degree

5 +5 2
+x+1) 2" (mod2).
Since + x + 1 is irreducible in Z2[x], by unique factorization we must
have

A(s) B(s) + x + 1)2"' + + 1 (mod 2).

Thus, if we write

A(s) = + + ao,
B(s) = + .. + b0,
then ao, b0 are odd and all the other coefficients
are even. Since f is monic, we may assume without loss of generality
124 4. Solutions to Advanced Problems

that = = 1; also, a0bo = f(O) = 1. but a0 > 0, > 0 as f has


no real roots, so a0 = b0 = 1.
Therefore,
([x2n+2"1] + [x2"1])(g(x)h(x))

+
( i=2"1
+ + +
EQ (mod 4)

as + is even.
But

+ [x2"'])(f(x))
=

and is odd by Lucas's theorem, so


([x2"+2"'] + [x2"')) (1(x)) 2 (mod 4),

a contradiction.
Hence f is irreducible.

Problem 99 [Iran 1998]


Let Ii, 12,13 R R be functions such that
aifi + a212 + a3f3
is monotonic for all a1, a2, a3 E
Prove that there exist Ci, c2, c3 c R, not all zero, such that

cifi(x) + c2f2(x) + c3f3(x) = 0

for all x E R.
Solution 99, Alternative 1
We establish the following lemma.
Lemma: Let f, g R p IR be functions such that f is nonconstant and
af + bg is monotonic for all a, b c R. Then there exists c R such that
g cf is a constant function.

Proofi Let s, t be two real numbers such that f(s) 1(t).


4. Solutions to Advanced Problems 125

Let
g(s)g(t)
- f(s) - f(t)
Let h1 = g d1f for some d1 E R.
Then h1 is monotonic. But

hi(s) h1(t) = g(s) g(t) d1(f(s) f(t)) = (f(s) f(t))(u d1).


Since f(s) f(t) 0 is fixed, the monotonicity of h1 depends only on
thesignofud1.
Since f is nonconstant, there exist xl,x2 c R such that 1(x1) f(x2).
Let
c= g(xl)g(x2)
f(xi) f(x2)
and h= g cf.
Then r = h(xi) = h(x2) and the monotonicity of h1 = d1f, for each
d1, depends only on the sign of c d1.
We claim that h = g cf is a constant function.
We prove our claim by contradiction.
Suppose, on the contrary, that there exists x3 c R such that h(xs) r.
Since f(xi) f(x2), at least one of f(xi) and f(x2) f(xi) is
true.
Without loss of generality, suppose that 1(x1) f(x3).
Let
c' = g(x1) g(x3)
f(xi) f(x3)
Then the monotonicity of h1 also depends only on the sign of c' d1.
Since h(x3) r = h(xi),

=c';
f(xi) f(x3)
hence cd1 c' d1.
So there exists some d1 such that h1 is both strictly increasing and de-
creasing, which is impossible.
Therefore our assumption is false and h is a constant function.
Now we prove our main result.
If fi f2, 13 are all constant functions, the result is trivial.
Without loss of generality, suppose that Ii is nonconstant.
126 4. Solutions to Advanced Problems

For a3 = 0, we apply the lemma to Ii and 12, so 12 = cf1 +d; for a2 0,


we apply the lemma to fi and 13, so = c'f1 + d'.
Here c, c', d, d' are constant.
We have

(c'd cd')f1 + d'f2 df3 = (c'd cd')fi + d'(cfi + d) d(c'fi + d') = 0.

If (c'dcd',d',d) (0,0,0), then let


(ci, c2, c3) = (c'd cd', d', d)

and we are done.


Otherwise, d = d' = 0 and 12,13 are constant multiples of Ii
Then the problem is again trivial.
Solution 99, Alternative 2
Define the vector
v(x) = (1i(x),12(x),13(x))
for x C R.
If the v(x) span a proper subspace of R3, we cart find a vector (ci, c2, c3)
orthogonal to that subspace, and then cifi(x) + c2f2(x) + c3f3(x) = 0
for all x R.
So suppose the v(x) span all of R3.
Then exist x1 < X2 < X3 c R such that v(xi), v(x2), v(x3) are
linearly independent, and so the 3 x 3 matrix A with = has
linearly independent rows.
But then A is invertible, and its columns also span R3.
This means we can find c2, c3 such that

= (0,1,0),

and the function cif1 +c2f2 +c3f3 is then not monotonic, a contradiction.
4. Solutions to Advanced Problems 127

Problem 100 [USAMO 1999 proposal, Richard Stong]


Let x1,x2,. . be variables, and let Y1,Y2,.
. be the sums of . .

nonempty subsets of
Let pk(xl,. be the kth elementary symmetric polynomial in
. . ,

the (the sum of every product of k distinct yj'S).


For which k and n is every coefficient of Pk (as a polynomial in x1,.. . ,

even?
For example, ifn = 2, then Y1,Y2,Y3 are +x2 and
Pi =Y1+Y2+Y32X1+2X2,
P2 Y1Y2+Y2Y3+Y3Y1
P3 = Y1Y2Y3 = +

Solution 100
We say a polynomial pk is even if every coefficient of Pk is even.
Otherwise, we say Pk is not even.
For any fixed positive integer n, we say a nonnegative integer k is bad
for n if k = for some nonnegative integer j.
We will show by induction on n that pk(xl, x2, is not even if and . ,

only if k is bad for ii.


For n = 1, = x1 is not even and k = 1 is bad for n = 1 as
k=1=212=r2.
Suppose that the claim is true for a certain n.
We now consider pk(x1,x2, . . .

Let 0k(yl, Y2,. .. Ys) be the elementary symmetric polynomial.


We have the following useful, but easy to prove, facts:

1.

2. For all 1 r
,y8) = i+j=k
> ,Yr)Uj(Yr+1, .

3. 0k(X+Y1,X+y2,...,X+Y&)
=
iI<22<"<ik

= .

Sl<82<<S,.
{si ,ik}
128 4. Solutions to Advanced Problems

Hence

=
i+j=k
+X2 + +

= , . ,

r.O

By the induction hypothesis, every term of Pr(Xi, x2 , xi,) is even un-

For such r, note that


2t
j - r) - -
isevenunlessjr=Oorj--r=2t.
Therefore, taking coefficients modulo 2,

,Xn)Pj(X1,X2,'
i+j=k

,Xn)p2n_2t(x1,X2,

By the induction hypothesis, the terms in the first sum are even unless

In the second sum, every term appears twice except the term

Pk/2(X1, ,

for k even.
By the induction hypothesis, this term is even unless k/2 = for
some 0 <v n, that is k = 2v+1

It follows that Pk(X1, x2, is even unless k = 2w for some


4. Solutions to Advanced Problems 129

Furthermore, note that the odd coefficients in

Pk(X1,X2,

occur for different powers of


Therefore, the condition that k is bad for n + 1 is also sufficient for

to be odd.
Our induction is complete.

Problem 101 [Russia 20001


Prove that there exist 10 distinct real numbers a1, a2,..., a10 such that
the equation

(x ai)(xa2)...(x a10) =(x-i--ai)(x-i-a2).(x+aio)


has exactly 5 different real roots.
Solution 101
We show that a2,. .. a10} =
, 6,. . 2} is a group of numbers
. ,

satisfying the conditions given in the problem.


The given equality becomes

(x 2)(x 1)x(x + 1)(x + 2)g(x2) 0,

where

g(u) = 2[((7+6++3)u2+
(7.65+7.6.4+...--5.4.3)u+763].
If g(u) = hasno real solutions, then g(x2) = 0 has no real solutions.
If u1 and u2 are real solutions of g(u) = 0, then u1 -F-u2 <0 and U1U2 > 0,
that is, both u1 and u2 are negative.
It follows again that g(x2) = 0 has no real solutions.
Our proof is complete.
GLOSSARY

Arithmetic-Geometric Mean Inequality (AMGM Inequality)


If a1, a2,.. . , are n nonnegative numbers, then

with equality if and only if al = a2 = =

Binomial Coefficient
The coefficient of in the expansion of (x + is

(n'\
= k!(nk)!

Cauchy-Schwarz Inequality
For any real numbers a1, a2,.. . , and b1, b2,.. ,

with equality if and only if and are proportional, i = 1,2,. .. , n.

Chebyshev Inequality

1. Let Xi, X2 . . , and yi, . . , be two sequences of real num-


bers, such that x1 X2 < and Yi y2 ... yn.
Then

+X2 + +Xn)(Yi +Y2+" +Yn) +X2y2 .

2. Let Xl,X2...,Xn and Y1,Y2,..,Yn be two sequences of real num-


bers, such that x1 x2 xi-, and Yt y2 ..
Then

+yn) Xlyl+X2y2+ +XnYn.


132 Glossary

De Moivre's Formula
For any angle a and for any integer n,
(cosa+ = cosna+isinna.

Elementary Symmetric Polynomials (Functions)


Given indeterminates x1, ..., Xn, the elementary symmetric functions
Si,. .. are defined by the relation (in another indeterminate t)
(t + x1) .. (t + = tTh + + + + Sn.

That is, Sk is the sum of the products of the taken k at a time. It


is a basic result that every symmetric polynomial in x1 can be
(uniquely) expressed as a polynomial in the and vice versa.

Fibonacci Numbers
Sequence defined recursively by F1 = F2 = 1, = + for all

n E N.

Jensen's Inequality
1ff is concave up on an interval [a, b] and A1, A2, ..., are nonnegative
numbers with sum equal to 1, then
Aif(xi)+A2f(x2)+-.. +).2X2+

for any x1 ,X2 in the interval [a. b]. If the function is concave
down, the inequality is reversed.

Lagrange's Interpolation Formula


Let Xo,Xi,. . ,x7, be distinct real numbers, and let YO.Y1,. ..
. be ar-
bitrary real numbers. Then there exists a unique polynomial P(x) of
degree at most n such that = i = 0. 1,.. . , n. This is the
polynomial given by

i=O
x0)

Law of Cosines
Let ABC be a triangle. Then
BC2 = AB2 + AC2 - 2AB . ACcosA.
Glossary 133

Lucas' Theorem
Let p be a prime; let a and & be two positive integers such that

a = + ak_lpkI + + a0, b = bkpk + + bip + b0,

where 0 b2 <p are integers for i = 0, 1 k. Then


(a'\ (ak\ (ak_1\ (aj\ (ao\ (modp).

Pigeonhole Principle
If n objects are distributed among k < n boxes, some box contains at
least two objects.

Root Mean Square-Arithmetic Mean Inequality (RMSAM In-


equality)
For positive numbers x1, X2,. . . ,

\/X?+X2++XkXl+X2++Xk
More generally, let a1, a2,.. . , be any positive numbers for which ai +
a2 + + = 1. For positive numbers x1, x2,... we define
,

= min{x1,x2,. .. ,xk},
Moc, =max{xl,x2,...,xk},
aj a2
0xl x2
= +
a non-zero real number. Then

<M8 M00
for s t.
134 Glossary

Triangle Inequality
Let z = a + bi be a complex number. Define the absolute value of z to
be
zI=
Let and /3 be two complex numbers. The inequality

is called the triangle inequality.


Let = and i9 = /9i +/32i, where /92 are real numbers.
Then
Vectors u = v = [/91,/92], and w = +192] form a
triangle with sides lengths and + /31
The triangle inequality restates the fact that the length of any side of a
triangle is less than the sum of the lengths of the other two sides.

Vieta's Theorem
Let x1, X2,. . . , be the roots of polynomial
P(x) = + +... + + a0.
where #0 and a0,a1, E C. Let 8k
. . . be the sum of the products
of the taken k at a time. Then

that is,

x1x2 + + = ;
x1x2 =

a
sin a
cos a
1

a
Glossary 135

addition and subtraction formulas:

sin(ab) =sinacosbcosasinb,
cos(ab)
tan(ab)= tan tan
a tan b
a tan b'
1

double-angle formulas:
sin2a = 2sinacosa,
cos 2a = cos2 a sin2 a = 2 cos2 a 1 = 1 2 sin2 a,
2 tan a
tan2a=
1tan2a'

triple-angle formulas:
sin3a = 3sina 4s1n3 a,
cos3a = 4cos3a 3cosa,
tan3a= 3tana 2 a
tan3
13tan a
half-angle formulas:
2 tan
sina=
1 2
1 tan2 2

1 tan2 2

a 2

cosa+cosb=
a+b ab

sin(a+b)
tana +tanb =
cosacosb'

difference-to-product formulas:
ab a+b
sin a sin b = 2 sin cos
136 Glossary

cos a cos b 2 sin


ab
sin
a+b
sin(a b)
tanatanb=
cos a cos b

product-to-sum formulas:

2sinacosb = sin(a + b) +sin(a b),


2cosacosb = cos(a + b) cos(a
2sinusinb = cos(a + b) + cos(a b).
FURTHER READING

1. Andreescu, T. Kedlaya, K.; Zeitz, P., Mathematical Contests


1995-1996: Olympiad Problems from around the World, with
Solutions, American Mathematics Competitions, 1997.
2. Andreescu, T. Kedlaya, K., Mathematical Contests 1996-1997:
Olympiad Problems from around the World, with Solutions,
American Mathematics Competitions, 1998.
3. Andreescu, T. Kedlaya, K., Mathematical Contests 1997-1998:
Olympiad Problems from around the World, with Solutions,
American Mathematics Competitions, 1999.
4. Andreescu, T. Feng, Z., Mathematical Olympiads: Problems and
Solutions from around the World, 1998-1999, Mathematical
Association of America, 2000.
5. Andreescu, T. Gelca, R., Mathematical Olympiad Challenges,
Birkhuser, 2000.
6. Barbeau, E., Polynomials, Springer-Verlag, 1989.
7. Beckenbach, E. F., Bellman, R., An Introduction to Inequalities,
New Mathematical Library, Vol. 3, Mathematical Association of
America, 1961.

8. Chinn, W. C., Steenrod, N. E., First Concepts of Topology, New


Mathematical Library, Vol. 27, Random House, 1966.
9. Cofman, J., What to Solve?, Oxford Science Publications, 1990.

10. Coxeter, H. S. M., Greitzer, S. L., Geometry Revisited, New


Mathematical Library, Vol. 19, Mathematical Association of
America, 1967.

11. Doob, M., The Canadian Mathematical Olympiad 1969-1993,


University of Toronto Press, 1993.

12. Engel, A., Problem-Solving Strategies, Problem Books in


Mathematics, Springer, 1998.
13. Fomin, D., Kirichenko. A., Leningrad Mathematical Olympiads
1987-1991, MathPro Press, 1994.
138 Further Reading

14. Fomin, D., Genkin, S., Itenberg, 1., Mathematical Circles,


American Mathematical Society, 1996.
15. Graham, R. L., Knuth, D. E., Patashnik, 0., Concrete
Mathematics, Addison-Wesley, 1989.
16. Greitzer, S. L., International Mathematical Olympiads, 1959-1977,
New Mathematical Library, Vol. 27, Mathematical Association of
America, 1978.
17. Grossman, I., Magnus, W., Groups and Their Graphs, New
Mathematical Library, Vol. 14, Mathematical Association of
America, 1964.

18. Kazarinoff, N. D., Geometric Inequalities, New Mathematical


Library, Vol. 4, Random House, 1961.
19. Klamkin, M., International Mathematical Olympiads, 1978-1985,
New Mathematical Library, Vol. 31, Mathematical Association of
America, 1986.
20. Klamkin, M., USA Mathematical Olympiads, 1972-1986, New
Mathematical Library, Vol. 33, Mathematical Association of
America, 1988.
21. Kiirschk, J., Hungarian Problem Book, Volumes I & II, New
Mathematical Library, Vols. 11 & 12, Mathematical Association of
America, 1967.

22. Kuczma, M., 144 Problems of the Austrian-Polish Mathematics


Competition 1978-1998, The Academic Distribution Center, 1994.
23. Larson, L. C., Problem-Solving Through Problems, Springer-Verlag,
1983.

24. Lausch, H., Bosch Giral, C., The Asian Pacific Mathematics Olym-
piad 1989-2000, AMT Publishing, Canberra, 2001.
25. Liu, A., Chinese Mathematics Competitions and Olympiads
1981-1993, AMT Publishing, Canberra, 1998.
26. Lozansky, E., Rousseau, C. Winning Solutions, Springer, 1996.
27. Ore, 0., Graphs and their uses, Random House, 1963.
28. Ore, 0., Invitation to number theory, Random House, 1967.
29. Sharygin, I. F., Problems in Plane Geometry, Mir, Moscow, 1988.
Further Reading 139

30. Sharygin, I. F., Problems in Solid Geometry, Mir, Moscow, 1986.

31. Shklarsky, D. 0, Chentzov, N. N; Yaglom, I. M., The USSR


Olympiad Problem Book, Freeman, 1962.
32. Slinko, A., USSR Mathematical Olympiads 1989-1992, AMT
Publishing, Canberra, 1997.
33. Soifer, A., Colorado Mathematical Olympiad: The first ten years,
Center for excellence in mathematics education, 1994.
34. Szekely, C. J., Contests in Higher Mathematics, Springer- Verlag,
1996.

35. Stanley, R. P., Enumerative Combinatorics, Cambridge University


Press, 1997.
36. Taylor, P. J., Tournament of Towns 1980-1984, AMT Publishing,
Canberra, 1993.
37. Taylor, P. J., Tournament of Towns 1984-1 989, AMT Publishing,
Canberra, 1992.
38. Taylor, P. J., Tournament of Towns 1989-1993, AMT Publishing,
Canberra, 1994.
39. Taylor, P. .1., Storozhev, A., Tournament of Towns 1993-1997,
AMT Publishing, Canberra, 1998.
40. Tomescu, I., Problems in Combinatorics and Graph Theory, Wiley
1985.

41. Vanden Eynden, C., Elementary Number Theory, McGraw-Hill,


1987.

42. Wilf, H. S., Generatingfunctionology, Academic Press, 1994.


43. Wilson, R., Introduction to graph theory, Academic Press, 1972.
44. Yaglom, 1. M., Geometric Transformations, New Mathematical
Library, Vol. 8, Random House, 1962.
45. Yaglom, I. M., Geometric Transformations II, New Mathematic
Library, Vol. 21, Random House, 1968.
46. Yaglom , I. M., Geometric Transformations III, New Mathemati
Library, Vol. 24, Random House, 1973.
47. Zeitz, P., The Art and Craft of Problem Solving, John Wiley &
Sons, 1999.
Titu Andreescu is
Director of the
A met jean
tics
Competitions,
serves as Head
Coach of the USA
international
Mathem atical
Olympiad (lMO)
Team, is Chair of the USA Mathematical
Olympiad Committee, and is Director of the
USA Mathematical Olympiad Summer
Program. Originally from Romania, Prof.
Andreescu received the Distinguished
Professor Award from the Romanian
Ministty of Education in 1983, then after
moving to the USA was awarded the Edyth
May Sliffe Award for Distinguished High
School Mathematics Teaching from the
MM in 1994. in addition, he received a
Certificate of Appreciation presented by the
President of the MM for This outstanding
service as coach of the USA Mathematical
Olympiad Program in prepanng the USA
team for its perfect performance in Hong
Xong at the 1994 1MO".

Zuming Feng
graduated with a
PhD from Johns
Hopkins University
with an
on Algebraic
Nuniber Theory
and Elliptic
Curves. He teaches
at Phillips Exeter
Academy. He also serves as a coach of the
USA international Mathematical Olympiad
(IMO) Team, is a member of the USA
Mathematical Olympiad Committee, and is
an assistant director of the USA
Mathematical Olympiad Summer Program.
He received the Edyth May Sliffe Award for
Distinguished High School Mathematics
THF AUSTRALIAN MATHEMATiCS TRUST Teaching from the MM in 1996.

ENRICHMENT SERIES 1S8N187642012X

Вам также может понравиться